Real Property

Réussis tes devoirs et examens dès maintenant avec Quizwiz!

A buyer bought a home from a real estate developer for $700,000. The buyer paid $100,000 of the purchase price herself. The buyer's employer provided $100,000 of the purchase price by giving the buyer a loan and taking a mortgage. The developer loaned $500,000 to the buyer to finance the remainder of the purchase price, and in return took a mortgage on the property. One week later, a bank obtained a judgment against the buyer for a delinquent credit card balance. The bank properly recorded its judgment as a lien against the property.Another month after that, the buyer incurred some extraordinary medical expenses, and asked the employer for another $100,000, which the employer provided and added onto the principal balance the buyer owed on the loan. Finally, six months later, the buyer asked the developer to change the terms of the loan, so that the buyer would have more time to pay. The developer and the buyer agreed that the buyer could have an additional five years to pay the balance of the loan in exchange for an increase in the principal of the loan. Shortly thereafter, the buyer lost his job and defaulted on all of his payments. The employer brought an action to foreclose its mortgage. All mortgages and liens were promptly and properly recorded. Regarding the distribution of the proceeds of an eventual sheriff's sale of the property, which of the following statements is true? A The bank is paid in full before the developer is paid in full. B The employer is paid in full before the bank receives any proceeds. C The developer is paid in full before the employer receives any proceeds. D The developer is paid in full before the employer is paid in full.

A. The bank will be paid in full before the developer is paid in full. Generally, the priority of mortgages is chronological. A number of other factors, however, may affect priority. Where a seller of property receives a mortgage as part of the purchase price, a purchase money mortgage results. Purchase money mortgages may also arise when a third party lends money to the buyer for the purchase of property and takes a mortgage on the property in return. In general, the seller's purchase money mortgage will take priority over the third-party purchase money mortgage. Purchase money mortgages, however, are subject to later liens by virtue of recording acts. In the case where a mortgage is modified by agreement between the parties, any increase in the debt resulting from the modification will be subject to a junior lien, even if the original mortgage itself had priority over the junior lien. In the same way, an optional (as opposed to an obligatory) advance that is made after the junior lien will have a lower priority than the junior lien. Again, this is the case even if the original mortgage is first in priority. Therefore, the distribution of sale proceeds in this case would be: (i) the original amount of the employer's purchase money mortgage, (ii) the bank's judgment, (iii) the $100,000 advance by the employer, and finally, (iv) the amount of the increase in the debt to the developer due to the agreed modification of the principal of the original loan. The original unmodified purchase money mortgage of the developer would remain on the land because it was senior to the mortgage being foreclosed (the employer's). (B) is incorrect because the employer would not be paid in full before the bank received payment. (C) is incorrect because the developer would initially receive only the increased amount of the debt according to the modified loan terms. Also, the developer would have to wait until the bank judgment was satisfied and the employer was paid in full before the developer's claim for the modification amount could be paid. (D) is incorrect because the developer would not be paid in full—its original purchase money mortgage would remain on the property after the foreclosure.

A deed generally must contain which of the following in order to be valid? A The grantor's acknowledgment. B The grantor's words of intent. C The grantee's signature. D The metes and bounds of the land.

B. A deed generally must contain the grantor's words of intent in order to be valid. A deed must demonstrate that the grantor intends to transfer realty (e.g., by using the word "grant"). However, no particular technical phrasing is necessary. A deed generally need not contain the grantor's acknowledgment in order to be valid. Before a deed can be recorded under most recording statutes, it must be acknowledged by the grantor before a notary public. However, the grantor's signature, without an acknowledgement, is sufficient for the deed itself to be valid. A deed generally need not contain the metes and bounds of the land in order to be valid. While a deed must identify the land, a metes-and-bounds description is only one of many ways property may be described. A description is sufficient if it provides enough information to identify the property in question (e.g., a street address, or a reference to a lot in a recorded subdivision plat). A deed generally need not contain the grantee's signature in order to be valid. Even if the deed contains covenants on the grantee's part, her acceptance of the deed is sufficient to make those covenants enforceable.

A deed delivered to a grantee who is dead at the time of delivery is: A Valid B Voidable C Void

C. A deed delivered to a grantee who is dead at the time of delivery is void. A deed is not effective to transfer an interest in real property unless it has been delivered. The grantee must actually exist at the time of delivery; otherwise, the deed is not valid (i.e., void). A void deed will be set aside by a court even if title purportedly has passed to a bona fide purchaser. A deed delivered to a grantee who is dead at the time of delivery is not voidable. In contrast with void deeds, voidable deeds will be set aside only if the property has not passed to a bona fide purchaser. Deeds considered voidable include those executed by incapacitated persons (e.g., minors) or obtained through fraud in the inducement, duress, undue influence, mistake, and breach of fiduciary duty. QUESTION ID: P0102A Additional Learning

May a grantee be bound by a covenant that does not appear in his deed or chain of title? response - incorrect A No, if the deed is a quitclaim deed. B No, if the deed contains a covenant against encumbrances. C Yes, if there is a common scheme for development and the grantee had notice of the covenant. D Yes, if the covenant touches and concerns the land.

C. A grantee may be bound by a covenant that does not appear in his deed or chain of title if there is a common scheme for development and the grantee had notice of the covenant. An equitable servitude is a covenant (i.e., a promise to do or not to do something on the land) that, regardless of whether it runs with the land at law, can be enforced in equity against successors to the burdened land who have notice of the covenant. Generally, equitable servitudes are created by covenants contained in a writing that satisfies the Statute of Frauds. However, in the absence of a writing, negative equitable servitudes may be implied if (i) there is a common scheme for development (i.e., a plan existing at the time sales of the subdivision parcels began that all parcels be developed within the terms of the negative covenant), and (ii) the grantee had actual, record, or inquiry notice of the covenant. Except as explained above, a grantee may not be bound by a covenant that does not appear in his deed or chain of title even if the covenant touches and concerns the land. The burden of an equitable servitude will run to successors in interest if: (i) the covenanting parties intended that successors in interest be bound by the covenant; (ii) the successor in interest has notice of the covenant; and (iii) the covenant touches and concerns the land (i.e., it benefits the covenantor and her successor in their use and enjoyment of the burdened land). In the absence of a writing, however, the servitude will not be enforced unless there is a common scheme for development as explained above. A grantee may be bound by a covenant that does not appear in his deed or chain of title even if the deed contains a covenant against encumbrances. This is a covenant contained in a general warranty deed assuring that there are neither visible encumbrances (e.g., easements) nor invisible encumbrances (e.g., mortgages) against the title or interest conveyed. The presence of this covenant does not affect the ability of a successor in interest to the covenantee to enforce an equitable servitude. A grantee also may be bound by a covenant that does not appear in his deed or chain of title even if the deed is a quitclaim deed. This type of deed conveys whatever interest, if any, the grantor has in the property. It does not affect the ability of a successor in interest to the covenantee to enforce an equitable servitude.

Which of the following is not an essential term of a land sale contract under the Statute of Frauds? A An identification of the parties to the contract. B A closing date. C A description of the property. D A price and manner of payment, if agreed upon.

B. A closing date is not an essential term of a land sale contract under the Statute of Frauds. The closing is when the parties tender performance of the contract, i.e., exchange the purchase price and deed. Matters incidental to the contract (e.g., furnishing of deeds, prorating of taxes, title insurance) can be determined by custom, and a closing date will be construed to be within a reasonable time. They need not appear in the writing nor even have been agreed upon. The essential terms of a land sale contract under the Statute of Frauds are: 1. A description of the property; 2. An identification of the parties to the contract; and 3. A price and manner of payment, if agreed upon. The Statute also requires the contract to be in writing and signed by the party to be charged.

A landowner conveyed his land to his wife, son, and daughter "as joint tenants with right of survivorship." The daughter then conveyed her interest to a friend. The wife subsequently executed a will devising her interest to the daughter. Then the son mortgaged his interest to a lender, who promptly and properly recorded the mortgage. The wife died, then the daughter's friend died, leaving a will that bequeathed her entire estate to the daughter. The daughter and the son survived. If the jurisdiction follows the title theory, who owns what interest in the land? A The lender owns the fee simple. B The lender and the daughter own unequal shares as tenants in common. C The son and the daughter own unequal shares as tenants in common. D The son and the daughter own equal shares as joint tenants.

B. The lender and the daughter own unequal shares as tenants in common. Creation of a joint tenancy requires four unities: (i) time (interests must vest at the same time); (ii) title (interests must be acquired by the same instrument); (iii) interest (interests must be of the same type and duration); and (iv) possession (interests must give identical rights to enjoyment). When property is held in joint tenancy by three or more joint tenants, a conveyance by one of them destroys the joint tenancy only as to the conveyor's interest. The other joint tenants continue to hold in joint tenancy as between themselves, while the grantee holds her interest as a tenant in common with them, because she does not share the unities of time or title with the joint tenants (i.e., her interest vested at a different time and was acquired by a different instrument). Here, the wife, the son, and the daughter owned the property as joint tenants. When the daughter conveyed her interest to the friend, the joint tenancy was severed as to the daughter's interest. At that point, the wife and the son each held one-third interests as joint tenants because, as between themselves, the four unities were preserved. The friend did not share the unities of time or title with the wife and the son. Thus, the friend took a one-third interest as a tenant in common rather than as a joint tenant. Because this jurisdiction follows the title theory, the son's mortgage also severed the joint tenancy. The minority of states following the title theory regard a mortgage as an actual transfer of title to the property. Thus, a mortgage by one joint tenant transfers the legal title of the joint tenant to the mortgagee (the lender). This action destroys the unity of title and severs the joint tenancy. On the other hand, in a lien theory state (majority), a mortgage is considered a lien on title--one joint tenant's execution of a mortgage on his interest does not, by itself, sever a joint tenancy until default and foreclosure proceedings have been completed. Here, when the son executed the mortgage, title was transferred to the lender, severing the joint tenancy again. At that point, the wife, the lender, and the friend each held one-third interests as tenants in common because each interest was acquired at a different time and by a different title. An interest in a tenancy in common is freely alienable by inter vivos and testamentary transfer, is inheritable, and is subject to claims of the tenant's creditors. The only "unity" involved is possession: Each tenant is entitled to possession of the whole estate. Thus, when the wife died, her interest passed under her will to the daughter. Likewise, when the friend died, her interest passed to the daughter. Thus, the lender holds a one-third interest as a tenant in common with the daughter, who holds the remaining two-thirds interest. (A) is incorrect because the daughter also has an interest in the property (see above). When one joint tenant dies, the right of survivorship operates to free the property of her interest; the surviving joint tenants retain an undivided right in the property. Although a will is inoperative as to joint tenancy property because the decedent's rights in the property evaporate at the instant of death, a will may effectively dispose of an interest held in a tenancy in common, as discussed above. Here, although the wife executed her will while the property was still held in joint tenancy, a will is ambulatory (effective only at death), and at her death the wife held her interest as a tenant in common. Thus, it passed to the daughter under the will. (C) and (D) are incorrect because, as explained above, the son has no interest.

As between two mortgages, what is the effect on the junior mortgage when the mortgagor accepts an advance of funds from the senior mortgagee? A The junior mortgage is given priority over the entire senior mortgage if the advance was optional B The junior mortgage is given priority over the advance if the advance was optional C The junior mortgage is given priority over the advance if the senior mortgagee was contractually obligated to make it D The junior mortgage is given priority over the entire senior mortgage if the senior mortgagee was contractually obligated to make it

B. When the mortgagor accepts an advance of funds from the senior mortgagee, the junior mortgage is given priority over the advance if the advance was optional. Priority among mortgages on the same real estate is normally determined by chronology: The earliest (i.e., senior) mortgage is first in priority, the next (i.e., junior) mortgage is second, and so on. Generally, if the mortgage obligates the mortgagee to make further advances of funds after the mortgage is executed, such advances will have the same priority as the original mortgage. However, if a junior mortgage is placed on the property and the senior mortgagee later makes an "optional" advance (i.e., one it was not contractually bound to make) while having notice of the junior mortgage, the advance will lose priority to the junior mortgage. Numerous states have reversed this rule by statute, but it remains the majority view. Thus, the junior mortgage is NOT given priority over the advance if the senior mortgagee was contractually obligated to make it. Furthermore, an advance would not jeopardize the priority of the entire senior mortgage itself; thus, the junior mortgage is NOT given priority over the entire senior mortgage, regardless of whether the advance was optional or the senior mortgagee was contractually obligated to make it.

Which of the following transfers creates a sublease from T to T2? response - correct A One year into a five-year tenancy for years, T transfers his interest "to T2 for four years; however, if T2 breaches the original lease terms, T may reenter and retake the premises" B Six months into a seven-month tenancy for years, T transfers his interest "to A for the balance of the leasehold term" C Two years into a four-year tenancy for years, T "assigns my entire interest to T2 for one year"

C. If two years into a four-year tenancy for years, T "assigns my entire interest to T2 for one year," the effect of the transfer is to create a sublease between T and T2. The label given by the parties does not determine whether a transfer is an assignment or a sublease. Rather, a complete transfer of a tenant's entire remaining lease term is an assignment, and a transfer retaining any part thereof is a sublease. Here, although T "assigned" his interest to T2, he transferred only one of the remaining two years of the lease. Thus, the transfer is a sublease rather than an assignment. If six months into a seven-month tenancy for years, T transfers his interest "to A for the balance of the leasehold term," the effect of the transfer is an assignment of the lease from T to T2 because it includes T's entire remaining lease term. If one year into a five-year tenancy for years, T transfers his interest "to T2 for four years; however, if T2 breaches the original lease terms, T may reenter and retake the premises," the effect of the transfer is an assignment of the lease from T to T2. T transferred the remaining four years of the lease to T2, and by the slight majority view, T's reservation of a right of reentry does not result in a sublease, but rather is still an assignment.

May a buyer obtain specific performance of an oral land sale contract? A Yes, provided the buyer has paid most of the purchase price B No, because an oral contract does not satisfy the Statute of Frauds C Yes, if the buyer has taken possession of and made substantial improvements to the land D No, because the buyer's remedy is damages

C. Yes, a buyer may obtain specific performance of an oral land sale contract if the buyer has taken possession of and made substantial improvements to the land. While land sale contracts must be memorialized in writing and signed by the party to be charged to be enforceable under the Statute of Frauds, courts in most states will enforce an oral contract in equity under the doctrine of part performance if the buyer has performed at least two of the following acts: 1. Taken possession of the land; 2. Made substantial improvements to the land; and/or 3. Paid all or part of the purchase price. Some courts will accept as part performance additional acts showing the buyer's detrimental reliance. A buyer might not obtain specific performance of an oral land sale contract even if the buyer has paid most of the purchase price. As explained above, most jurisdictions require at least two acts of part performance. For an oral land sale contract, the buyer's remedy is NOT damages. Only specific performance is available in equity under the doctrine of part performance. Although an oral contract does not satisfy the Statute of Frauds, a court may award specific performance if the buyer shows sufficient acts of part performance. Two theories support this remedy: (i) the buyer's acts unequivocally evidence an oral contract; and (ii) the buyer's detrimental reliance estops the seller from asserting the Statute of Frauds as a defense.

If an occupier initially has the true owner's permission to enter the land, may she acquire title to the land by adverse possession? A Yes, unless the occupier believes she is on her own land B No, because the statute of limitations will not begin to run C No, because an adverse possessor must lack the true owner's permission to be on the land D Yes, if the occupier communicates hostility

D. Yes, an occupier who initially has the true owner's permission to enter the land may acquire title to the land by adverse possession if the occupier communicates hostility and satisfies the other elements of adverse possession. To establish title by adverse possession, the occupier must show: (i) An actual entry giving exclusive possession that is (ii) Open and notorious, (iii) Adverse (hostile), and (iv) Continuous throughout the statutory period. If the occupier enters with the owner's permission, her possession may become adverse only once she makes it clear to the owner that she is claiming hostilely. This can be done by explicit notification, by refusing to permit the true owner to come onto the land, or by other acts inconsistent with the original permission. The occupier's state of mind is irrelevant to adverse possession, which means that it does not matter whether the occupier believes she is on her own land, knows she is trespassing on someone else's land, or has no idea who owns the land. While it is true that an adverse possessor must lack the true owner's permission to be on the land, a subsequent communication of hostility may cause initially permissive possession to become adverse, as explained above. The statute of limitations WILL begin to run if an occupier who initially had the true owner's permission to enter the land communicates hostility, as explained above.

For which type of security interest in land does the debtor transfer title to a third party acting on behalf of the lender? A Deed of trust B Installment land contract C Absolute deed D Equitable mortgage

A. A deed of trust is a security interest in land by which the debtor (i.e., the trustor) transfers title to the land to a third party (i.e., the trustee), such as the lender's lawyer or a title insurance company, acting on behalf of the lender (i.e., the beneficiary). In the event of default, the lender instructs the trustee to foreclose the deed of trust by selling the property. An equitable mortgage exists if a court concludes that a grantor transferred an absolute deed to serve as security for an obligation. If the court so determines, the grantee must foreclose by judicial action, as with any other mortgage. The court will consider: (i) The existence of a debt or promise of payment by the grantor; (ii) The grantee's promise to return the land if the debt is paid; (iii) Whether the amount advanced to the grantor was much lower than the value of the property; (iv) The degree of the grantor's financial distress; and (v) The parties' prior negotiations. An installment land contract is a security interest in land in which the debtor (i.e., the buyer) contracts with the seller to pay for the land in regular installments until the full contract price has been paid, plus interest. Only then will the seller transfer legal title to the buyer. The contract may contain a forfeiture clause providing that the seller may cancel the contract upon default, retain all money paid, and retake possession of the land.

Which of the following deeds is most likely to be set aside as against a bona fide purchaser? A A forged deed B A deed executed by a debtor with intent to defraud a creditor C A deed executed under duress D A deed executed by a minor grantor

A. A forged deed is most likely to be set aside as against a bona fide purchaser. A forged deed is void, and deeds that are void will be set aside by a court even if the property has passed to a bona fide purchaser. In contrast, deeds executed by a minor grantor or executed under duress are voidable. Voidable deeds will be set aside only if the property has not passed to a bona fide purchaser. A deed executed by a debtor with intent to defraud a creditor may be set aside by the grantor's creditors as a fraudulent conveyance. Under the Uniform Fraudulent Transfer Act, which nearly all states have adopted, a conveyance is fraudulent if the grantor/debtor actually intended to hinder, delay, or defraud a creditor; or if the grantor (i) did not receive a reasonably equivalent value in exchange for the transfer and (ii) was insolvent or became insolvent as a result of the transfer. However, such a conveyance will not be set aside as against a grantee who took in good faith and paid reasonably equivalent value (i.e., a bona fide purchaser). QUESTION ID: P0104A Additional Learning

When may a mortgagor redeem her land in equity? A Before the foreclosure sale. B Only during the foreclosure sale. C After the foreclosure sale. D At any time, unless waived in the mortgage itself.

A. A mortgagor may redeem her land in equity before the foreclosure sale. At any time prior to the foreclosure sale—i.e., this right does not exist only during the foreclosure sale—the mortgagor has the right to redeem the land or free it of the mortgage by paying off the amount due, plus interest. If the mortgage or note contained an acceleration clause, which permits the mortgagee to declare the full balance due in the event of default, the full balance must be paid in order to redeem. A mortgagor may not redeem her land in equity after the foreclosure sale. However, about half the states provide a statutory right to redeem—distinct from the equitable right discussed above—for some fixed period after the foreclosure sale has occurred (e.g., six months or one year). While a mortgagor may redeem her land in equity at any time before the foreclosure sale, this right cannot be waived in the mortgage itself. Doing so is known as "clogging the equity of redemption" and is prohibited. However, the right can be waived later, for consideration.

Which of the following statements regarding specific performance of a land sale contract is true? A Both the buyer and the seller generally are entitled to specific performance B If the seller cannot convey marketable title, the buyer may not obtain specific performance C Specific performance is available only to the buyer D Specific performance is available only to the seller

A. Both the buyer and the seller generally are entitled to specific performance of a land sale contract. A court of equity will order a seller to convey title if the buyer tenders the purchase price. The remedy at law, damages, is deemed inadequate because land is unique. Courts also generally will award specific performance for the seller if the buyer is in breach, although a few courts will do so only if the property is especially unique (e.g., not if a developer is selling a house in a large subdivision of similar houses). The seller's ability to recover in equity is sometimes explained as necessary for mutuality of remedy. In either case, specific performance is NOT available only to the seller or only to the buyer. If the seller cannot convey marketable title, the buyer MAY obtain specific performance of the land sale contract with an abatement of the purchase price in an amount reflecting the title defect.

A homeowner borrowed $50,000 from a bank, secured by a mortgage on his home. Shortly thereafter, the homeowner sold his home to a buyer for $70,000 by a deed containing a recital signed by both parties that title passed "subject to" the bank's mortgage, "which obligation grantee expressly assumes." The buyer paid the homeowner $20,000, took possession of the house, and began making monthly payments of principal and interest to the bank. A few years later, a chemical manufacturing firm built a huge sulfur processing plant just down the road from the home, which caused the house to immediately decline in value to $35,000. Subsequently, the buyer stopped making the monthly payments to the bank. The bank exercised its contractual right of nonjudicial foreclosure and sold the house at a public auction for $34,000. The bank then brought suit against the homeowner and the buyer for $14,000, the difference between the proceeds of the foreclosure sale and the $48,000 principal remaining due on the original loan to the homeowner. The jurisdiction does not bar deficiency judgments. Against whom should the bank be granted a judgment for $14,000? A Both the homeowner and the buyer. B Only the homeowner. C Only the buyer. D No one.

A. Both the homeowner and the buyer are liable for the deficiency. If a sale of foreclosed property does not bring enough to satisfy the mortgage debt, the mortgagee/lender can bring a personal action against the mortgagor/debtor for the deficiency (as long as the jurisdiction does not bar deficiency judgments). When the mortgagor sells the mortgaged property and gives a deed, the grantee takes subject to the mortgage, which remains on the land. If the grantee does not sign an agreement to assume the mortgage, he does not become personally liable on the loan, and the original mortgagor remains personally liable. If the grantee does sign an assumption agreement, however, the lender is considered a third-party beneficiary of the agreement, and hence may recover from the assuming grantee, who is primarily liable, or the original mortgagor, who is secondarily liable. Here, the buyer signed the recital providing for the assumption, so she will be personally liable on the loan. Therefore, (A) is correct, and (B) is incorrect. (C) is incorrect because the homeowner, the original mortgagor, did not extinguish his own personal liability on the loan by obtaining the assumption agreement from the buyer. He remains secondarily liable as a surety. Thus, the bank may sue the homeowner on the original mortgage agreement. (Note that while the bank may obtain a judgment against both of them, its maximum recovery will be the $14,000 deficiency.) (D) is incorrect because the facts indicate that the jurisdiction does not bar deficiency judgments.

Which of the following acts will terminate an easement? A Condemnation of the servient estate. B Use of the easement beyond its legal scope. C Nonuse of the easement for the statutory period. D Voluntary destruction of the servient estate.

A. Condemnation of the servient estate will terminate an easement. The easement holder may be entitled to compensation for the value lost. Use of the easement beyond its legal scope will not terminate an easement. Instead, the easement is surcharged, and the servient owner may sue to enjoin the use. Nonuse of the easement for the statutory period will not terminate an easement. An easement can be extinguished by the easement holder's physical act of abandonment (e.g., erection of a permanent structure over the easement). However, mere nonuse, even for a long period of time, is insufficient to constitute an abandonment of the easement. To terminate the easement, the nonuse must be combined with other evidence of intent to abandon it. Voluntary destruction of the servient estate (e.g., tearing down a building to erect a new one) will not terminate an easement. On the other hand, involuntary destruction of the servient estate (e.g., by fire or flood) will extinguish the easement.

Which of the following interests in property are subject to the Rule Against Perpetuities? A Contingent remainders, executory interests, and vested remainders subject to open B Reversions, options, and class gifts C Executory interests, rights of entry, and powers of appointment D Contingent remainders, possibilities of reverter, and rights of first refusal

A. Contingent remainders, executory interests, and vested remainders subject to open are subject to the Rule Against Perpetuities. The Rule Against Perpetuities provides that certain interests in property are void if there is any possibility, however remote, that they may vest more than 21 years after some life in being at the creation of the interest. The Rule applies to the following interests in property: (i) contingent remainders; (ii) executory interests; (iii) class gifts (even if vested remainders); (iv) options and rights of first refusal; and (v) powers of appointment. Future interests in the grantor (i.e., reversions, possibilities of reverter, and rights of entry) are not subject to the Rule Against Perpetuities. Thus, contingent remainder, possibilities of reverter, and rights of first refusal is incorrect because it includes possibilities of reverter. Executory interests, rights of entry, and powers of appointment is incorrect because it includes rights of entry. Reversions, options, and class gifts is incorrect because it includes reversions.

Which of the following is NOT required for the benefit of a real covenant to run to successors in interest? A There is horizontal privity between the original covenanting parties. B There is vertical privity between the covenantee and her successor in interest. C The covenant touches and concerns the land. D The covenanting parties intended that successors in interest be benefitted by the covenant.

A. Horizontal privity between the original covenanting parties is not required for the benefit of a real covenant to run to successors in interest. A real covenant is a written promise to do or not to do something on the land. The benefit of the covenant will run to successors in interest if: 1. The covenanting parties intended that successors in interest bebenefitted by the covenant; 2. There is vertical privity between the covenantee and her successor n interest; and 3. The covenant touches and concerns the land (i.e., it benefits the covenantee and her successor in their use and enjoyment of the benefited land). For the burden of a real covenant to run to successors in interest, vertical privity requires that the successor in interest to the covenanting party hold the entire durational interest held by the covenantor at the time he made the covenant. On the other hand, vertical privity for the running of the benefit is satisfied when the successor in interest holds any possessory estate, even a lesser estate.

Which of the following statements is correct regarding covenants against assignment or sublease? A If a landlord consents to one transfer that violates a covenant against assignment or sublease, he waives his right to avoid future transfers. B If a tenant transfers her interest in violation of a covenant against assignment or sublease, the transfer is void. C A covenant against assignment or sublease is an unreasonable restraint on alienation. D A covenant against assignment prevents the tenant from subleasing her interest.

A. If a landlord consents to one transfer that violates a covenant against assignment or sublease, he waives his right to avoid future transfers. This is the Rule in Dumpor's Case. The landlord may reserve the right to avoid future transfers, but such reservation must take place at the time of granting consent. A covenant against assignment or sublease is NOT an unreasonable restraint on alienation. All jurisdictions permit and enforce such covenants. A covenant against assignment does NOT prevent the tenant from subleasing her interest. Covenants against assignment or sublease are strictly construed against the landlord. Thus, a covenant prohibiting assignment does not prohibit subleasing and vice versa. If a tenant transfers her interest in violation of a covenant against assignment or sublease, the transfer is NOT void. However, the landlord usually may terminate the lease under the lease terms or a statute or sue for damages.

In order to prevail over a prior grantee under a race-notice statute, when must a subsequent bona fide purchaser record? A Before the prior grantee records. B A bona fide purchaser will prevail over a prior grantee under a race-notice statute without recording. C Before she learns of the prior grant. D Before she takes possession of the property.

A. In order to prevail over a prior grantee under a race-notice statute, a subsequent bona fide purchaser ("BFP") must record before the prior grantee records. A race-notice statute is a recording act that alters the common law rule of "first in time, first in right" to protect a subsequent BFP ( i.e., one who gives valuable consideration and lacks notice of the prior conveyance). However, to obviate questions about the time of delivery and to induce parties to record promptly, race-notice statutes only protect BFPs who are first to record. A subsequent BFP need not record before she learns of the prior grant. The notice requirement of BFP status is measured from the time of the subsequent grantee's receiving her conveyance. Thus, a BFP who lacked notice at the time of the conveyance, then learned of the prior interest, and then recorded, can still prevail over a prior interest—provided the BFP records before the prior grantee records. A subsequent BFP under a race-notice statute need not record before she takes possession of the property. Rather, she must record before a prior grantee records. The date that the subsequent BFP takes possession is immaterial for purposes of prevailing over a prior grantee. A bona fide purchaser will NOT prevail over a prior grantee under a race-notice statute without recording. A race-notice statute contains the recording requirement discussed above. By contrast, under a notice statute, a subsequent BFP need not record in order to prevail over a prior grantee who failed to record. A notice statute requires only that the subsequent purchaser have no actual or constructive (i.e., record or inquiry) notice at the time of the conveyance. This is not the case under a race-notice statute.

Which of the following is a future covenant for title? A Covenant of warranty B Covenant against encumbrances C Covenant of seisin D Covenant of right to convey

A. The covenant of warranty is a future covenant for title. A general warranty deed contains covenants for title through which the grantor warrants against title defects created by herself and prior titleholders. The usual covenants for title include present covenants, which can be breached only at the time of conveyance; and future covenants, which can be breached only upon eviction (i.e., interference with the possession of the grantee or her successors by someone with better title). Through the covenant of warranty, the grantor agrees to defend the grantee's title from any third party's lawful or reasonable claims of title and to compensate the grantee for any related loss. Because this covenant cannot be breached until a third party interferes with possession, it is a future covenant. The covenant of seisin is a present covenant for title. Through it, the grantor warrants that she has the estate or interest she purports to convey (i.e., both title and possession) at the time of the grant. The covenant against encumbrances is a present covenant for title. Through it, the grantor warrants that there are no encumbrances (e.g., easements, profits, or mortgages) against the title or interest conveyed. The covenant of right to convey is a present covenant for title. Through it, the grantor warrants that she has the power and authority to make the grant (i.e., she has title or is the titleholder's authorized agent).

Which of the following correctly states a presumption regarding the delivery of a deed? A A grantor's acknowledgment of a deed before a notary and subsequent recordation constitutes a valid delivery B A grantee's return of a deed is a reconveyance to the grantor C Acceptance is presumed unless the grantee is an infant or incompetent D A grantee rejects a deed unless she expressly accepts it

A. The following correctly states a presumption regarding the delivery of a deed: A grantor's acknowledgment of a deed before a notary and subsequent recordation constitutes a valid delivery. A deed will not transfer an interest in land unless it has been delivered by the grantor and accepted by the grantee. Delivery is presumed if the deed is: (i) handed to the grantee, or (ii) acknowledged by the grantor before a notary and recorded. A grantee's return of a deed is NOT a reconveyance to the grantor. Title passes to the grantee upon effective delivery, and returning the deed to the grantor has no effect. To effect a reconveyance, the grantee must execute and deliver a new deed. A grantee does NOT reject a deed because she failed to expressly accept it. In most states, the opposite is true—acceptance is presumed if the conveyance is beneficial to the grantee, regardless of whether the grantee knows of it. In other states, acceptance is presumed only where the grantee knows of the grant and fails to reject it. In all states, acceptance IS presumed if the grantee is an infant or an incompetent. QUESTION ID: P0108A Additional Learning

A developer and an investor had been in the real estate business for many years. Because of their long-standing relationship, the developer and the investor, neither of whom was an attorney, often dispensed with certain legal formalities when dealing with each other, thus saving the costs of lawyers' fees and other attendant expenses. The investor owned a parcel of land that the developer was interested in, and she offered to buy it from him for $50,000. The investor accepted the developer's offer, and the parties agreed on June 15 as the closing date. The developer handed the investor a check for $2,500 with "earnest money" written in the memo, and they shook hands on their deal. A few weeks before closing, the developer called the investor and told him she had changed her mind about purchasing the land because of a a sudden economic downturn in the area. The investor appeared at the developer's office on June 15 with the deed to the land in his hand. The developer refused to tender the balance due, and the investor sued the developer for specific performance. Will the investor prevail? A No, because the agreement does not comply with the Statute of Frauds and is, therefore, unenforceable. B No, but the court will allow the investor to keep the $2,500 earnest money as damages. C Yes, because the $2,500 payment constituted part performance of the contract. D Yes, because the developer and the investor had established a course of dealing.

A. The investor will not succeed in a suit for specific performance because the agreement is unenforceable under the Statute of Frauds. Under the Statute of Frauds, a land sale contract is unenforceable unless it is in writing and signed by the party to be charged. The Statute of Frauds requires the writing to contain all essential terms of the contract, which are: (i) a description of the property, (ii) identification of the parties to the contract, (iii) the price and manner of payment, and (iv) the signature of the party to be charged. Here, the agreement between the investor and the developer concerns the sale of land; thus, the agreement must be in writing to comply with the Statute of Frauds. The only writing mentioned in the facts is the check given to the investor by the developer. This check contains neither a description of the property that is the subject of the agreement nor the price and manner of payment. Thus, the check is not a writing sufficient to satisfy the Statute of Frauds. Consequently, the agreement is unenforceable, and the investor will not prevail. (B) is incorrect because, if there is no enforceable agreement, there can be no "breach" of the agreement, for which breach the investor would be entitled to damages. Therefore, the investor may not keep the earnest money as damages. (C) is incorrect. Under the doctrine of part performance adopted by some states, a court may grant specific performance of a contract despite the absence of a writing if there has been payment of the purchase price. Even under this view, however, the developer's payment of $2,500 out of a total price of $50,000 will not constitute sufficient performance to remove this agreement from the purview of the Statute of Frauds. (D) is incorrect because "course of dealing" (i.e., a sequence of previous conduct between the parties that may be regarded as establishing a common basis of their understanding) may be used to explain or supplement the terms of a written contract under the Uniform Commercial Code ("UCC"). This question does not involve the sale of goods, so the UCC is inapplicable. Furthermore, here there is no written agreement, the terms of which can be explained or supplemented by showing a course of dealing between the developer and the investor. Although the developer and the investor often dispensed with legal formalities as a cost-saving measure, this "course of dealing" will not confer validity on their oral agreement for the sale of land.

Two partners bought a commercial building from an owner. They paid cash for the building and took title as joint tenants with right of survivorship. Several years later, the first partner executed a mortgage on the building to secure a personal loan to a bank. The second partner had no knowledge of the mortgage to the bank. The state in which the commercial building is located recognizes the lien theory of mortgages. The first partner died before paying off his loan. He left all of his property by will to his daughter, his only heir. Who has title to the commercial building? response - incorrect A The second partner has title free and clear of the mortgage. B An undivided one-half is held by the second partner free and clear of the mortgage, and the other one-half is held by the daughter, subject to the mortgage. C An undivided one-half is held by the second partner and the other one-half by the daughter, with both halves subject to the mortgage. D The second partner has title to the entire property, with an undivided one-half being subject to the mortgage.

A. The second partner has title free and clear of the mortgage. When the partners bought the property, they took title as joint tenants with right of survivorship. If the joint tenancy continued until the first partner's death, then the property would pass immediately on death to the second partner. Because the second partner did not sign the mortgage, she would not be subject to it, regardless of whether she knew about it. The key to answering this question is to know whether execution of the mortgage by the first partner caused a severance of the joint tenancy. If it did cause a severance, then the first partner's one-half would not pass to the second under right of survivorship but instead would pass to the first's estate, and thus would go to the daughter by will. Whether a mortgage creates a severance or not depends on whether the state follows the lien theory or the title theory of mortgages. Lien theory means no severance; title theory means severance. Because this is a lien theory state (majority rule on the MBE), there was no severance; thus, the joint tenancy remained intact. On the first partner's death, the joint tenancy ended and the first partner's interest instantly passed to the second partner. The first partner's estate got nothing; hence, the daughter could get nothing. (B) would be the correct answer if the execution of the mortgage had created a severance of the joint tenancy. The severance would have changed the joint tenancy to a tenancy in common. The second partner would keep her one-half, free of the mortgage she did not sign, and the daughter would inherit the first partner's one-half, subject to the mortgage the first partner signed. Because we are told this is a lien theory state, there was no severance when the mortgage was executed so the joint tenancy remained intact until the first partner's death. (C) is incorrect. Not only is the joint tenancy unsevered so that it remains intact to give title to the entire property to the second partner, but also under no circumstances could the second partner be held liable for a mortgage she did not enter into. (D) is incorrect. When the first partner died, the property passed free and clear to the surviving joint tenant. The mortgage signed by the first partner did not sever the joint tenancy because this is a lien theory state. The mortgage can be held only against the property the first partner has; when the first partner died, the right of survivorship operated to end the first partner's interest and automatically vested it in the survivor.

When a watercourse or body of water forms the boundary of a parcel of real property, accretion causes the legal boundary to change due to __________. A The slow and imperceptible deposit of soil B The sudden, perceptible change of the watercourse C The encroachment of the body of water

A. When a watercourse or body of water forms the boundary of a parcel of real property, accretion causes the legal boundary to change due to the slow and imperceptible deposit of soil. The slow erosion of the bank of a river or stream serving as a boundary results in the owner losing title to that area. Similarly, the slow deposit of soil belongs to the owner of the abutting land. Where accretion builds up in an irregular pattern over the lands of several adjacent property owners, courts determine title to it by (i) extending each owner's property line out into the water, or (ii) dividing up the newly formed land in proportion to the owners' interests in the adjoining lands. In contrast, avulsion is the sudden, perceptible change of a watercourse serving as a boundary. Avulsion does not change the legal boundaries, which remain where they were even if this results in a landowner who had river access finding himself landlocked. Similarly, the legal boundary line is not changed by the encroachment of the body of water. When property is encroached upon by a body of water (e.g., a lake enlarges), previously fixed boundary lines do not change and ownership rights are not affected. The boundary lines can be proven even if the land is completely under water. QUESTION ID: P0106A Additional Learning

A person whose interest in land gives him the right to use someone else's land independent of his ownership or possession of his own tract holds: A An easement appurtenant B An easement in gross C A license D A servient tenement

B. A person whose interest in land gives him the right to use someone else's land independent of his ownership or possession of his own tract holds an easement in gross. An easement gives the holder the right to use a tract of land but no right to possess it. The land burdened by the easement right is called the servient tenement. An easement appurtenant, by contrast, benefits its holder in his physical use or enjoyment of his own tract of land. The land benefitted by the easement is called the dominant tenement. Unlike an easement, a license is not an interest in land, but is merely a privilege to go upon another's land.

When is parol evidence admissible to show oral conditions on the delivery of a deed? A If the deed was given directly to the grantee B If the deed was placed in escrow C Always D never

B. Parol evidence is admissible to show oral conditions on the delivery of a deed if the deed was placed in escrow. A grantor may deliver a deed by giving it to a third party (i.e., an escrow) who is instructed to give it to the grantee. Parol evidence generally is admissible to show the conditions and terms on which the deed was placed in escrow, even though the deed itself is unconditional. Parol evidence is not admissible for this purpose if the deed was given directly to a grantee. In most jurisdictions, parol evidence is not admissible to show that the delivery of a facially unconditional deed given directly to a grantee was in fact subject to a condition. Note, however, that parol evidence generally will be admitted to show that a grantor did not intend to deliver a deed or that the deed was intended merely as security for an obligation. QUESTION ID: P0118B Additional Learning

A landowner conveyed her parcel of land to "my brother and my sister jointly, with right of survivorship." Shortly thereafter, the brother was in an automobile accident. The driver of the other vehicle sued the brother on a theory of negligence, and obtained a judgment in the amount of $250,000. Because the brother did not have insurance or enough cash to satisfy the judgment, the driver levied on the brother's interest in the land. What interest will the driver most likely take? response - incorrect A None, because the brother's interest in the land cannot be partitioned. B An undivided one-half interest, regardless of whether the brother and the sister's title to the land is construed as a joint tenancy or a tenancy in common. C An undivided one-half interest, assuming the brother and the sister's interest is construed as a tenancy in common and not a joint tenancy. D A contingent right of survivorship that will vest if the brother survives the sister.

B .The driver will get an undivided one-half interest in the land regardless of the status of the brother and the sister's title. A joint tenancy is a concurrent estate with a right of survivorship, while a tenancy in common does not have a right of survivorship. At common law, the conveyance here would qualify as a joint tenancy because the unities of time, title, interest, and possession are present in the conveyance. Although under modern law a joint tenancy must be created with specific language or else it will be presumed to be a tenancy in common, the conveyance here still would probably qualify as a joint tenancy, even though it did not use the words "joint tenancy," because it contained the "right of survivorship" language. However, regardless of whether the estate is characterized as a joint tenancy or a tenancy in common, one tenant's interest may be transferred without the consent of the other tenant, and a creditor may levy on the interest. In most jurisdictions, a lien against one joint tenant's interest does not sever the joint tenancy until the lien holder proceeds to enforce it by foreclosure. At that point, the purchaser at the foreclosure sale will hold the property as a tenant in common with the other tenant, but will still have an undivided one-half interest in the property unless and until he brings an action to partition the estate. (A) is incorrect because both joint tenancies and tenancies in common may be subject to partition. (In contrast, tenancies by the entirety cannot be terminated by involuntary partition.) (C) is incorrect because, as discussed above, a joint tenant may validly convey or encumber his interest in the property. (D) is incorrect because the driver does not have a contingent interest; she has a present lien on the brother's interest that can be enforced immediately by foreclosure, which would sever the joint tenancy.

Against whom may a grantee invoke the doctrine of estoppel by deed? A The original grantor and subsequent bona fide purchasers only B The original grantor only C The original grantor and any successor to the grantor's after-acquired title

B. A grantee may invoke the estoppel by deed doctrine against the original grantor only. Under the doctrine of estoppel by deed, if a grantor purports to convey an estate that he does not then own, his subsequent acquisition of title to the property automatically inures to the benefit of that grantee. In other words, the grantor impliedly covenants that he will convey title upon its acquisition and is estopped to deny that he acquired title on the grantee's behalf. The majority holds that this is a personal estoppel; thus, title inures to the benefit of the grantee only as against the original grantor and NOT any successor to the grantor's after-acquired title. Consequently, if the original grantor transfers his after-acquired title to a subsequent bona fide purchaser ("BFP") —i.e., one who pays valuable consideration and lacks notice of the prior conveyance, the BFP gets good title and the grantee has no rights against the BFP. Note also that in most states, if the original grantee records the deed, a subsequent purchaser will not be charged with record notice (and thus could be a BFP) because the grantor's earlier deed would be outside the subsequent purchaser's chain of title. QUESTION ID: P0117B Additional Learning

Which of the following acts will NOT sever a joint tenancy? A Inter vivos conveyance by one joint tenant. B Testamentary disposition by one joint tenant. C In a title theory state, the execution of a mortgage by one joint tenant. D Suit for partition by one joint tenant.

B. A testamentary disposition by one joint tenant will not sever a joint tenancy. A will devising a joint tenant's interest to another is inoperative as to joint tenancy property because when the testator dies ( i.e., when the will becomes effective), his rights in the joint tenancy property are extinguished, and the will has no effect on them. A suit for partition by one joint tenant will sever a joint tenancy. The court will either divide the tract into parcels (partition in kind) or sell the property and divide the proceeds among the joint tenants in accordance with their ownership interests (partition by sale). An inter vivos conveyance by one joint tenant will sever a joint tenancy. The transferee takes the interest as a tenant in common and not as a joint tenant. In a title theory state, the execution of a mortgage by one joint tenant will sever a joint tenancy. In states following the lien theory, a mortgage is regarded as a lien on title, and one joint tenant's execution of a mortgage on her interest does not by itself cause a severance. But in states following the title theory, a mortgage is regarded as a transfer of title, and the transfer destroys the unity of title and severs the joint tenancy.

To buy a house, an investor secured a $10,000 mortgage from a bank. The bank promptly and properly recorded its mortgage. Subsequently, the investor financed certain improvements to the house with a $2,000 mortgage on the land from a finance company. The finance company promptly and properly recorded its mortgage. Before the investor made a payment on either mortgage, the federal government announced that it would begin storing nuclear waste products in the area. The value of property, including the investor's house, plummeted. The investor did not pay either the bank or the finance company, and the bank brought a proper action to foreclose, notifying both the investor and the finance company. A buyer bought the house at the foreclosure sale for $6,000, which was its fair market value. There are no special statutes in the jurisdiction regarding deficiency judgments. What does the investor owe? A $5,000 to the bank and $1,000 to the finance company. B $4,000 to the bank and $2,000 to the finance company. C Nothing to the bank and $2,000 to the finance company. D $4,000 to the bank and nothing to the finance company.

B. Absent any anti-deficiency statutes, the investor remains personally liable to pay for any shortfall arising from the foreclosure sale. Proceeds from the sale are used to satisfy the loan that was foreclosed first. Hence, all of the proceeds ($6,000) went to the bank. Thus, the investor must pay the balance still due the bank ($4,000) and the entire amount of the finance company's mortgage ($2,000), which is terminated by the foreclosure of the senior mortgage. (A) is wrong because foreclosure sales are not allotted proportionally between senior and junior interests. (C) is wrong because foreclosure does not extinguish the underlying debt. (D) is wrong because the finance company's mortgage does not remain on the land after foreclosure of the senior mortgage; hence, the investor is liable for that debt as well.

What does it mean for a grantee to assume a mortgage? A The grantee becomes a surety for the original mortgagor. B The grantee becomes primarily liable to the lender. C The grantee institutes foreclosure proceedings. D The grantee takes out an additional mortgage on the property.

B. For a grantee to assume a mortgage means the grantee becomes primarily liable to the lender. When a mortgagor conveys mortgaged property, the grantee takes the land subject to the mortgage. A grantee who signs an assumption agreement promises to pay the mortgage loan, thus becoming personally and primarily liable to the lender. The original mortgagor becomes secondarily liable as a surety. Assumption of a mortgage does not mean the grantee becomes a surety for the original mortgagor. The assuming grantee becomes primarily liable to the lender, and the original mortgagor becomes secondarily liable as a surety. Assumption of a mortgage does not mean the grantee institutes foreclosure proceedings. Foreclosure is a process that terminates the mortgagor's interest in the property. Generally, the property is sold in a foreclosure sale to satisfy the mortgage debt. The grantee who assumes a mortgage promises to pay the mortgage loan; thus, if the grantee defaults, foreclosure proceedings may be brought against him. Assumption of a mortgage does not mean the grantee takes out an additional mortgage on the property. As explained above, a grantee who signs an assumption agreement promises to pay the original mortgage loan, thus becoming primarily liable to the lender.

A landowner owned a large parcel of land in a rural area. He built his home on the northern half of the property, and developed a large orchard of fruit trees on the southern portion. A county road ran in front of the northern portion. To service his orchard, the landowner built a driveway directly from the county road across the northern portion of the property to the orchard. To provide electricity to his house, the landowner ran an overhead power line across the orchard property to hook up to the only available electric power pole located on the far southern side of the property. Subsequently, the landowner conveyed the northern parcel to his brother and the southern parcel to his daughter, who said that she did not mind having the power line on the property. Recently, the brother has begun parking his car on the driveway, thus blocking the daughter's access to the southern parcel. Finding no recorded document granting an easement for the power line, the daughter has decided to remove it. If the brother is successful in preventing the daughter from removing the power line, what is the likely reason? response - incorrect A The daughter knew that the power line ran across the land when she accepted the deed from the landowner. B The brother's alternative access to power is much less convenient and would cost 100 times as much. C The daughter told the landowner that she did not mind having the power line on the property. D The daughter is acting in retaliation against the brother for blocking the driveway, and not in any good faith belief that she has the right to remove the power line.

B. If the brother is successful in preventing the daughter from removing the power lines, it will be because the brother's alternative access to power is much less convenient and would cost 100 times as much as the current arrangement. This helps to prove that there was an easement implied by operation of law ("quasi-easement"). An easement may be implied if, prior to the time the tract is divided, a use exists on the "servient part" that is reasonably necessary for the enjoyment of the "dominant part," and a court determines that the parties intended the use to continue after division of the property. To give rise to an easement, a use must be apparent and continuous at the time the tract is divided. In this case, the landowner used the servient part of his property (the southern parcel) to run an overhead power line to the dominant part of his property (the northern parcel). Overhead wires are clearly visible and would be readily discoverable on reasonable inspection. The lines are, therefore, apparent. The use must also be reasonably necessary. Whether a use is reasonably necessary depends on many factors, including the cost and difficulty of the alternatives. This use was reasonably necessary to the enjoyment of the dominant parcel because electricity is important to the enjoyment of the property, and the cost (100 times as much) and difficulty of the alternatives are excessive. Thus, the fact that the use of the southern parcel is reasonably necessary would bolster the brother's case. (A) is wrong because the daughter's actual knowledge is irrelevant. The daughter need not be aware of the use; it need only be shown that the use was apparent (see above). (C) is similarly wrong. Oral statements made to the grantor after the northern parcel had been conveyed have little effect. They show the daughter's knowledge, but as discussed above, that has little relevance with respect to an implied easement. (D) is wrong because the daughter's motive for removing the power line is also irrelevant. If no easement is established, the daughter may remove the lines for whatever reason she likes. If, however, the requirements for an implied easement are satisfied, the daughter may not remove the lines regardless of how good her reasons are.

In most states, a buyer who has paid only part of the purchase price under an installment land contract: A Is protected by recording acts as though he had purchased the property outright B Is protected by recording acts only to the extent of payments made C Does not qualify as a purchaser and thus is not protected by recording acts

B. In most states, a buyer who has paid only part of the purchase price under an installment land contract is protected by recording acts only to the extent of payments made. Notice and race-notice recording acts protect bona fide purchasers ("BFPs") from prior unrecorded conveyances of the same property. A BFP is a purchaser who takes land without notice of a prior instrument and pays valuable consideration. In a dispute between a subsequent purchaser under an installment land contract and a prior claimant of the same land, the court may: 1. Award the contract purchaser a share of the property equal to the proportion of payments made as a tenant in common with the prior claimant;2. Award the land to the prior claimant, but give the contract purchaser a lien on the property to the extent of the amount paid; or3. Award the land to the contract purchaser, but give the prior claimant a lien on the property to the extent of the balance still owed. Thus, in most states, a purchaser who has paid only part of the purchase price under an installment land contract DOES qualify as a purchaser and is somewhat protected by the recording acts (as stated above), but he is NOT protected by recording acts to the extent he would be had he purchased the property outright. QUESTION ID: P0115B Additional Learning

Regarding the delivery and acceptance of a deed, which of the following will courts generally NOT presume? A A grantee accepts a deed if the conveyance would benefit her. B A grantee's return of a deed is a reconveyance to the grantor. C A grantee accepts a deed if she is a minor. D A grantee's possession of a deed means it has been delivered.

B. Regarding the delivery and acceptance of a deed, courts will not presume that a grantee's return of a deed is a reconveyance to the grantor. A deed will not transfer an interest in land unless it has been delivered by the grantor and accepted by the grantee. Title passes to the grantee upon effective delivery, and returning the deed to the grantor has no effect. To effect a reconveyance, the grantee must execute and deliver a new deed. Courts generally will presume that a grantee's possession of a deed means it has been delivered. Unless the grantor clearly expressed his intent that title pass to the grantee without physical delivery, the grantor's continued possession of the deed raises a presumption of nondelivery. Conversely, the grantee's possession of a properly executed deed raises a presumption of delivery. Regarding acceptance of a deed, courts generally will presume that a grantee accepts a deed if the conveyance would benefit her. In most states, acceptance is presumed if the conveyance is beneficial to the grantee, regardless of whether the grantee knows of it. In other states, acceptance is presumed only where the grantee knows of the grant and fails to reject it. Courts also will presume that a grantee accepts a deed if she is a minor. In all states, acceptance is presumed if the grantee is an infant or an incompetent.

A father purchased a tract of land, financing a large part of the purchase price by a loan from a bank that was secured by a mortgage on the land. A provision in the mortgage agreement, which had an acceleration clause, provided that a defaulting borrower waives his right to redeem once foreclosure proceedings have started. The bank properly recorded its mortgage. Several years later, the father needed money to send his twin daughters to college, so he obtained a loan from a credit union, also secured by a mortgage on the land. The credit union properly recorded its mortgage. The following year, the father became ill and was unable to make payments to either the bank or the credit union due to his high medical bills. The balance on the loan from the bank was $75,000, and the balance on the credit union loan was $25,000. The bank instituted foreclosure proceedings in a jurisdiction that provides a statutory right of redemption. The day before the judicial sale, the father inherited $100,000 from his aunt. He quickly contacted the bank and offered to pay off both loans in full. The bank refused because it was hoping to buy the now valuable property at the judicial sale. If the father seeks to force the bank to accept his offer, will he likely prevail? A Yes, because the jurisdiction has a statutory right of redemption. B Yes, because equity requires a creditor to accept such an offer. C No, because his agreement with the bank waived his right to redeem once foreclosure proceedings started. D No, because he lost all of his rights in the property when he defaulted on the loan.

B. The father will win because he is exercising his equity of redemption rights. The equity of redemption gives the borrower the right to free the land of the mortgage by paying off the amount due, plus any accrued interest, at any time prior to the foreclosure sale. If the borrower has defaulted on a mortgage with an acceleration clause, he must pay the full balance in order to redeem. Here, the father's offer to pay both the bank's and the credit union's loans is adequate to redeem the land. (A) is incorrect because a statutory right of redemption, recognized in about half the states, gives the borrower a right to redeem for the foreclosure price after the foreclosure sale. (C) is incorrect because this right to redeem cannot be waived in the agreement establishing the security interest. This would be "clogging the equity of redemption." However, the right can be waived later for consideration. (D) is similarly incorrect because a defaulting debtor does not lose the equity of redemption.

A landlord rented an art studio to an artist. Under the terms of the signed, written, two-year lease, the artist agreed to pay the landlord $1,000 per month and to assume responsibility for all necessary repairs. After the first year of the lease, the artist assigned the balance of his lease to a sculptor. The landlord approved the sculptor as a tenant and accepted two rent payments from her, and then the landlord sold the building to an investor. The sculptor had made two payments to the investor when an electrical fire broke out in the studio, injuring the sculptor. The fire was caused by faulty wiring. The landlord was aware that there was a dangerous wiring problem when he leased the property to the artist. But when the landlord discovered how costly repairs would be, he decided it would be more profitable to sell the property than to repair it. The problem was not easily discoverable by anyone other than an expert electrician, and the landlord did not tell the artist, the sculptor, or the investor about the problem. The sculptor sues to recover damages for her injuries. From whom can the sculptor recover? response - correct A The investor, because she breached the implied warranty of habitability. B The landlord, because he failed to disclose a latent defect. C The artist, because the artist is considered the sculptor's landlord. D No one, because the covenant to repair runs with the land, and the sculptor is bound by it.

B. The landlord is liable for the sculptor's injuries because he failed to disclose a latent defect. If, at the time the lease is entered into, the landlord knows of a dangerous condition that the tenant could not discover upon reasonable inspection, the landlord has a duty to disclose the dangerous condition. Failure to disclose the information about the condition results in liability for any injury resulting from the condition. Because the landlord knew of the dangerous electrical problem at the time he leased the premises to the artist and did not disclose it to either the artist or the sculptor, he is liable for any injuries resulting from that condition. (A) is wrong for two reasons: (i) the implied warranty of habitability does not apply to commercial leases; and (ii) even if this were a residential lease, it is doubtful that the investor would be liable for a condition of which she had no knowledge or notice. (C) is wrong because it describes the relationship between the artist and the sculptor as though there had been a sublease, when the facts clearly state that the artist assigned the balance of the lease to the sculptor. If a tenant sublets the premises (i.e., the tenant retains part of the remaining term), the tenant is the landlord of the sublessee. The sublessee cannot sue or be sued by the landlord. However, if there has been an assignment (i.e., the tenant makes a complete transfer of the entire term remaining), the assignee is substituted for the original tenant and can sue or be sued by the landlord. The original tenant's relationship to the assignee is at most that of a surety. Here, because the artist transferred the balance of his lease to the sculptor, there was an assignment and thus the artist cannot be considered the sculptor's landlord. (D) is wrong because, as stated above, the sculptor can recover from the landlord. The statement that the covenant to repair runs with the land and binds the sculptor is true, but the landlord's failure to disclose a dangerous preexisting condition renders the landlord liable for the sculptor's injuries despite the covenant; i.e., the sculptor's covenant does not relieve the landlord of his tort liability.

A landowner and his neighbor purchased adjoining undeveloped lots. After both built homes on their respective lots, the landowner suggested to the neighbor that a common driveway be built where the two lots joined. The neighbor agreed. The landowner and the neighbor split the cost of constructing the driveway and entered into a written agreement to equally share the costs of its upkeep and maintenance. The agreement was recorded in the county recorder's office. Two years later, the neighbor built a new driveway located entirely on his lot. The common driveway, which the landowner continued to use but which the neighbor no longer used, began to deteriorate. The landowner asked the neighbor for money to maintain the common driveway, but the neighbor refused to contribute. Three years later, the neighbor conveyed his lot to a friend. The friend entered into possession and used only the driveway built by the neighbor. By this time, the common driveway had deteriorated badly and contained numerous potholes. The landowner asked the friend to pay half of what it would take to repair the common driveway. The friend refused. The landowner repaired the driveway and sued the friend for 50% of the cost of repairs. Will the landowner prevail? A Yes, because easements run with the land. B Yes, because the agreement between the landowner and the neighbor was recorded. C No, because the neighbor abandoned use of the easement. D No, because the landowner is not in privity of contract with the friend.

B. The landowner will prevail because recording the agreement gave the friend constructive notice, thus preventing her from claiming the protection of the recording act as a defense to enforcement of the covenant. A covenant at law will run with the land and be enforceable against subsequent grantees if: (i) the contracting parties intended it to run; (ii) there is privity of estate between the original promisor and promisee (horizontal privity), as well as between the promisor and his successor (vertical privity); (iii) the covenant touches and concerns the property; and (iv) the burdened party has notice of the covenant. If common driveway owners agree to be mutually responsible for maintaining the driveway, the burdens and benefits of these covenants will run to successive owners of each parcel. The implied cross-easements for support satisfy the horizontal privity requirement because they are mutual interests in the same property. Each promise touches and concerns the adjoining parcel. So here, where the friend is in vertical privity with the neighbor (holding the same interest he held) and has constructive notice, she will be bound by the agreement to maintain the driveway. Although easements appurtenant, such as those involved in these facts, pass with the transfer of the estates involved, (A) is wrong because the easements are not at issue here. The easements involved are implied cross-easements for support and allow each party the right to enter the other's property when using the driveway. The issue here is the accompanying covenant to pay for the maintenance of the driveway. (C) is wrong for the same reason. Whether the easement has been abandoned does not affect the enforceability of the separate covenant. (D) is wrong because privity of estate, not privity of contract, is required for the burden of the covenant to run.

A retiree purchased a rustic cabin on a small plot of land near the center of a landowner's large parcel of land. The deed to the land, which the landowner delivered to the retiree for fair consideration, did not specifically grant an easement over the landowner's property to reach the public highway bordering her land. There were two means of access to the cabin from the public roads: a driveway from the county road on the south, and a private road from the highway on the east. The landowner told the retiree that he could use the private road from the highway. Twice during his first two years at the cabin, the retiree took the driveway from the county road instead; at all other times he used the private road. At the end of his second year at the cabin, the retiree began reading tarot cards to supplement his retirement income. He had a steady stream of clients coming to his home at all hours of the day and night. Most of the clients came in on the driveway from the county road, which ran close to the landowner's home. The landowner objected, and told the retiree that neither he nor his clients had any right to use that driveway and that they must use the private road from the highway. The retiree refused, and he and his clients continued to use the driveway from the county road for three years. Finally, the landowner began blocking off the driveway from the county road. The retiree brought suit to enjoin this practice. The prescriptive period in this jurisdiction is five years. Who will most likely prevail? response - incorrect A The landowner, because the tarot business has changed the nature of the use of the easement by necessity. B The landowner, because she may select the location of the easement. C The retiree, because he has a valid easement by necessity in the driveway from the county road. D The retiree, because he has acquired an easement by prescription in the driveway from the county road.

B. The landowner will prevail in a suit because she, as the holder of the servient estate, has the right to choose the location of an easement by necessity. An easement by necessity arises when the owner of a tract of land sells a part of the tract and by this division deprives one lot of access to a public road or utility line. The owner of the servient parcel has the right to locate the easement, provided the location is reasonably convenient. The landowner has chosen the private road from the highway; thus, the retiree has no right to use the driveway from the county road. Both (A) and (C) are incorrect because the retiree has no easement by necessity in the driveway. As stated above, the owner of the servient parcel (the landowner) has located the easement in the private road; thus, no easement in the driveway exists. When the owner of an easement uses it in a way that exceeds its legal scope (i.e., the easement is surcharged), the servient landowner may enjoin the excess use and possibly collect damages. If the easement by necessity had been located in the driveway, the excess use from the tarot business could have been the basis for the court's ruling in the landowner's favor. However, as stated above, the easement is in the private road from the highway. (D) is incorrect because the retiree's use has not been continuous for the five-year period. To acquire an easement by prescription, the use must be: (i) open and notorious, (ii) adverse, and (iii) continuous and uninterrupted for the statutory period. Continuous adverse use does not mean constant use. Periodic acts that put the owner on notice of the claimed easement fulfill the requirement. In this case, however, two uses in the first two years would not be sufficient to put the landowner on notice that the retiree intended to claim an easement in the driveway. Therefore, the retiree has not acquired a prescriptive easement in the driveway from the county road.

What is the result of the sudden, perceptible change of a watercourse that serves as a boundary to real property? A The newly formed land is divided in proportion to the owners' interests in the adjoining lands B No change to the riparian landowners' property rights C The boundary line shifts with the watercourse D No change to property rights unless a riparian landowner would find himself landlocked

B. The sudden, perceptible change of a watercourse serving as a boundary results in no change to the riparian landowners' property rights. This process is known as avulsion, and it does not shift the boundary line with the watercourse even if this results in a riparian landowner finding himself landlocked. In contrast, accretion refers to a change in a watercourse due to the slow and imperceptible deposit of soil. Accretion does change the legal boundary; the slow erosion of the bank results in the owner losing title to that area, and the slow deposit of soil belongs to the owner of the abutting land. Where accretion builds up in an irregular pattern over the lands of several adjacent property owners, courts determine title to it in a "just and equitable manner." This means either (i) each owner's property line is extended out into the water, or (ii) the newly formed land is divided in proportion to the owners' interests in the adjoining lands. QUESTION ID: P0106B Additional Learning

When a grantee's successor is ejected by a third party, the original grantor who conveyed the general warranty deed is not breaching __________. A the covenant of warranty B the covenant of seisin C any of the usual covenants for title in a general warranty deed D the covenant for quiet enjoyment

B. When a grantee's successor is ejected by a third party, the original grantor who conveyed the general warranty deed is not breaching the covenant of seisin. A general warranty deed contains covenants for title through which the grantor warrants against title defects created by himself and prior titleholders. Through the covenant of seisin, the grantor warrants that he has the estate or interest he purports to convey (i.e., both title and possession) at the time of the grant. The covenant of seisin can be breached only at the time of the initial conveyance; thus, it does not run to the grantee's successors. On the other hand, the original grantor who conveyed the general warranty deed is breaching some of the usual covenants for title in a general warranty deed. Among the usual covenants in a general warranty deed are future covenants, which run with the land. Thus, a grantor could be liable for breaching one of these covenants for title, as explained below, if the grantee's successors are disturbed in possession. When a grantee's successor is ejected by a third party, the original grantor who conveyed the general warranty deed is breaching the covenant for quiet enjoyment. Through this covenant, a grantor warrants that the grantee will not be disturbed by a third party's lawful claim of title to the property. This covenant runs with the land to the grantee's successors and can be breached multiple times. The lawful ejectment of the grantee or her successor would constitute a breach. The original grantor also is breaching the covenant of warranty. Through this covenant, the grantor agrees to defend the grantee's title from any third party's lawful or reasonable claims of title, and to compensate the grantee for any related loss. This is generally considered similar to the covenant for quiet enjoyment, and its benefit also runs with the grantee's estate. QUESTION ID: P0110 Additional Learning

When a watercourse or body of water forms the boundary of a parcel of real property, the legal boundary line is __________. A changed by a sudden change in the watercourse B changed by a slow, imperceptible change in the watercourse C not changed by movement of the water alone D changed by the encroachment of the body of water

B. When a watercourse or body of water forms the boundary of a parcel of real property, the legal boundary line is changed by a slow, imperceptible change in the watercourse. When a river or stream serves as a boundary, the slow erosion of the bank results in the owner losing title to that area. Similarly, the slow deposit of soil—i.e., accretion—belongs to the owner of the abutting land. Where accretion builds up in an irregular pattern over the lands of several adjacent property owners, courts determine title to it by (i) extending each owner's property line out into the water, or (ii) dividing up the newly formed land in proportion to the owners' interests in the adjoining lands. When a watercourse or body of water forms the boundary of a parcel of real property, the legal boundary line is NOT changed by a sudden change in the watercourse. When a river or stream serves as a boundary, the sudden, perceptible change of the watercourse—i.e., avulsion—does not change property rights. The boundaries remain where they were, even if this results in a landowner who had river access finding himself landlocked. When a watercourse or body of water forms the boundary of a parcel of real property, the legal boundary line is NOT changed by the encroachment of the body of water. When property is encroached upon by a body of water (e.g., a lake enlarges), previously fixed boundary lines do not change and ownership rights are not affected. The boundary lines can be proven even if the land is completely under water. When a watercourse or body of water forms the boundary of a parcel of real property, the legal boundary line CAN be changed by movement of the water alone. Through accretion or erosion, discussed above, a slow, imperceptible change in a watercourse will extend or retract the property line. QUESTION ID: P0106 Additional Learning

In a residential subdivision, will a commercial builder be bound by a residential-use restriction that was omitted from his deed? A No, because there is no written restrictive covenant in the deed to the builder's lot B Yes, if the builder had inquiry notice of a common scheme for development C Yes, if the builder is in horizontal privity with the developer D No, unless the builder had actual notice of restrictive covenants in the deeds to other lots

B. Yes, a commercial builder will be bound by the restriction if the builder had inquiry notice of a common scheme for development. An equitable servitude is a covenant (i.e., a promise to do or not do something on the land) that, regardless of whether it runs with the land at law, can be enforced in equity against assignees of the burdened land who have notice of it. Generally, equitable servitudes are created by covenants contained in a writing that satisfies the Statute of Frauds. However, in the absence of a writing, reciprocal negative servitudes may be implied if: 1. There is a common scheme for development (i.e., a plan existing at the time sales of the subdivision parcels began that all parcels be developed within the terms of the negative covenant); and 2. The grantee had actual, record, or inquiry notice of the covenant. Thus, the builder may be bound without actual notice of restrictive covenants in the deeds to other lots. In a residential subdivision, the builder would be on inquiry notice of a common scheme for development if the neighborhood appeared to conform to common restrictions. Thus, the builder would be bound by the residential-use restriction. Even though there is no written restrictive covenant in the deed to the builder's lot, the restriction may be enforced as a reciprocal negative servitude, discussed above. To be bound by the restriction, the builder need NOT be in horizontal privity with the developer. Horizontal privity requires that the original parties to a real covenant shared some interest in the land independent of the covenant at the time they entered it (e.g., as grantor and grantee). Horizontal privity is required to enforce the burden of a real covenant at law, but it is not required to enforce the burden of an equitable servitude.

May a tenant remove a chattel that the tenant affixed to the leased premises? A No, because chattels affixed to the leased premises become the property of the landlord B Yes, if removal occurs before termination of the lease and leaves no damage to the premises C No, unless the landlord and tenant expressly agreed that the chattel would be considered a fixture D Yes, because chattels affixed to the leased premises remain the property of the tenant

B. Yes, absent an agreement to the contrary, a tenant may remove a chattel that the tenant affixed to the leased premises if removal occurs before termination of the lease and leaves no damage to the premises. A fixture is a chattel that has been so affixed to the realty that it has ceased being personal property and has become part of the realty. At early common law, chattels affixed to the leased premises became the property of the landlord and thus could not be removed from the premises by the tenant. Today, a tenant may remove annexed chattels before the termination of the tenancy if doing so causes no damage to the premises (or any damage done by removing them is repaired by the tenant). Chattels affixed to the leased premises do NOT always remain the property of the tenant. Chattels the tenant affixes to leased premises may become the landlord's property (i.e., fixtures) under certain circumstances, such as where the tenant fails to remove the chattels before the end of the lease term, removal of the chattels will substantially damage the premises, or the landlord and tenant agreed that the chattels were intended to become fixtures. A tenant may NOT remove an affixed chattel if the landlord and tenant expressly agreed that the chattel would be considered a fixture. An agreement between the landlord and tenant is controlling as to whether the chattel annexed to the premises was intended to become a fixture. Thus, if the landlord and tenant agree that an annexation is not a fixture, the tenant will be permitted to remove it before the end of the lease term. However, if the landlord and tenant agree that an annexation is a fixture, it becomes the landlord's property.

Which of the following would not make title to land unmarketable? A Evidence that a prior grantor lacked capacity to convey the property B A significant variation in the description of property from one deed to the next C The existence of a mortgage on which the statute of limitations has run D The defective execution of a prior deed in the chain of title

C .The existence of a mortgage on which the statute of limitations has run would not make title to land unmarketable. Every land sale contract contains an implied covenant that the seller will provide marketable title at closing. While it need not be perfect title, it must not present the buyer with an unreasonable risk of litigation. Generally, this means an unencumbered fee simple with good record title. Title may be unmarketable because of a defect in the chain of title. Examples include a significant variation in the description of the land from one deed to the next, the defective execution of a prior deed in the chain of title that thus fails to meet the requirements for recordation, and evidence that a prior grantor lacked capacity to convey the property. Many courts hold that an ancient lien or mortgage on the record will not render title unmarketable if the seller has proof of its satisfaction or the statute of limitations on the claim would have run under any possible circumstance, including tolling for disabilities.

Which of the following deeds is the least likely to be set aside by a court? A A deed that was forged B A deed that was never delivered C A deed executed by a debtor with intent to defraud a creditor D A deed the grantor signed because he was told it was a promissory note

C. A deed executed by a debtor with intent to defraud a creditor is the deed least likely to be set aside by a court. Voidable deeds will be set aside only if the property has not yet passed to a bona fide purchaser. Under the Uniform Fraudulent Transfer Act, which nearly all states have adopted, a conveyance is fraudulent and may be set aside by the grantor's creditors if the grantor/debtor actually intended to hinder, delay, or defraud a creditor; or if the grantor (i) did not receive a reasonably equivalent value in exchange for the transfer and (ii) was insolvent or became insolvent as a result of the transfer. However, such a deed will not be set aside as against any grantee who took in good faith and paid reasonably equivalent value. In contrast with voidable deeds, void deeds will be set aside by a court even if the property has passed to a bona fide purchaser. Deeds considered void include those that are forged, were never delivered, or were obtained by fraud in the factum (meaning the grantor was deceived and did not realize that he was executing a deed—e.g., a deed the grantor signed because he was told it was a promissory note). QUESTION ID: P0104B Additional Learning

A fee simple subject to an executory interest is an estate that: A Continues after the happening of a stated event until the grantor exercises her power of termination B Continues after the happening of a stated event until the third party exercises his power of termination C Automatically divests in favor of a third party on the happening of a stated event D Automatically terminates on the happening of a stated event and reverts to the grantor

C. A fee simple subject to an executory interest is an estate that automatically divests in favor of a third party (rather than the grantor) on the happening of a stated event. It is created by the same language used to create a fee simple determinable (e.g., "for so long as," "while," "during," or "until") or a fee simple subject to a condition subsequent ( e.g., "upon condition that," "provided that," "but if," or "if it happens that"), but rather than automatically reverting to the grantor on the happening of a stated event (fee simple determinable) or continuing after the happening of a stated event until the grantor exercises her power of termination (fee simple subject to a condition subsequent), it automatically divests in favor of a third party on the happening of a stated event. A fee simple subject to an executory interest is not an estate that continues after the happening of a stated event until the third party exercises his power of termination. An estate that continues on the happening of a stated event until the grantor exercises her power of termination (right of entry) is a fee simple subject to a condition subsequent. A right of entry can be created only in favor of the grantor and her heirs. If a similar interest is created in favor of a third party, it is called an executory interest. However, unlike a right of entry, the third party need not "exercise" his executory interest; on the happening of the stated event, the estate automatically divests in his favor. The common law did not recognize a future interest created in a third party that would vest in possession only upon the discretionary exercise of a right or power by the third party. A fee simple subject to an executory interest is not an estate that automatically terminates on the happening of a stated event and reverts to the grantor. As explained above, such an estate is a fee simple determinable. A fee simple subject to an executory interest is not an estate that continues after the happening of a stated event until the grantor exercises her power of termination. As explained above, such an estate is a fee simple subject to a condition subsequent.

Which of the following is NOT a nonpossessory interest in land? A Easement B Real covenant C License D Profit

C. A license is not a nonpossessory interest in land. A license is merely a privilege to go upon another's land; it is not an interest in land. An easement is a nonpossessory interest in land. The holder of an easement has the right to use another's land, but has no right to possess and enjoy the land. A profit is a nonpossessory interest in land. The holder of a profit has the right to go upon another's land and take the soil or a substance of the soil (e.g., minerals, timber), but has no right to possess and enjoy the land. A real covenant is a nonpossessory interest in land. A real covenant is a written promise to use or not to use land in a certain manner, and does not confer a right to possess the land on the covenantee.

Which of the following always is required in order to transfer legal title to real property? A A competent grantor B Manual transfer of a written deed C A living grantee D Consideration

C. A living grantee always is required in order to transfer legal title to real property. Property cannot be conveyed to a grantee who does not exist. Thus, a deed to a grantee who is dead at the time of delivery is void. Manual transfer of a written deed is not always required in order to transfer legal title to real property. While a delivery is necessary for a deed to pass title, delivery need not consist of a physical handing over of the deed. Delivery merely requires some act coupled with a present intent to pass title. For example, a deed recorded by the grantor is presumed to have been delivered. Moreover, if the grantor is physically disabled, words alone, or even nods or gestures, may accomplish a delivery. A competent grantor is not always required in order to transfer legal title to real property. While title may be transferred through operation of law, land typically is conveyed by deed. To be valid, a deed must be in writing, identify the land and the parties, and be signed by the grantor. A deed that is defective may be classified as either void or voidable. Voidable deeds include those executed by one without capacity (e.g., because of minority or insanity). While a void deed can be set aside by the court at any time, a voidable deed will be set aside only if the property has not yet passed to a bona fide purchaser. Moreover, a court may appoint a guardian or conservator to execute a deed on behalf of an incompetent grantor. Consideration is not required in order to transfer legal title to real property. A grantor can convey real property by inter vivos gift if (i) he has donative intent, (ii) the deed is delivered, and (iii) the deed is accepted. Consideration is not necessary. QUESTION ID: P0102 Additional Learning

Which of the following parties would be entitled to prevail against a prior transferee under "notice" and "race-notice" statutes? A A donee of the land B A judgment creditor C A mortgagee for value D One who took the land by specific bequest

C. A mortgagee for value would be entitled to prevail against a prior transferee under "notice" and "race-notice" statutes. Notice and race-notice recording acts protect bona fide purchasers ("BFPs") from prior unrecorded conveyances of the same property. A BFP is a purchaser who takes land without notice of a prior instrument and pays valuable consideration. Donees, heirs, and devisees are not BFPs because they do not give value for their interests; i.e., they are not purchasers. Thus, a donee of the land and one who took the land by specific bequest would not be entitled to protection under the recording acts. In contrast, mortgagees for value ( i.e., those who loan substantial money in return for a mortgage on the land) are treated as "purchasers," either expressly by the recording act or by judicial classification. A judgment creditor would not be entitled to prevail against a prior transferee under "notice" and "race-notice" statutes. In nearly all states, a plaintiff who obtains a money judgment can obtain, by statute, a judgment lien on the defendant's real estate. The majority of cases hold that the judgment lienor is not protected either because (i) he is not a BFP because he did not pay value for the judgment, or (ii) the judgment attaches only to property "owned" by the defendant, and not to property the defendant has previously conveyed away, even if that conveyance was not recorded.

Horizontal privity exists between: A A party burdened under a real covenant and any party seeking to enforce the covenant B An original party to a real covenant and her successor in interest C Parties to a real covenant who shared an independent interest in the land at the time they entered the covenant D The original parties to a real covenant, regardless of their relationship

C. Horizontal privity exists between parties to a real covenant who shared an independent interest in the land at the time they entered the covenant. A real covenant is a written promise to do or not do something on the land. The burden of the covenant will run with the land if: 1. The covenanting parties intended that successors in interest be bound by the covenant; 2. The successor in interest has notice of the covenant; 3. There is horizontal privity between the original covenanting parties; 4. There is vertical privity between the covenantor and her successor in interest; and 5. The covenant touches and concerns the land. Horizontal privity requires that the original covenanting parties shared some interest in the land independent of the covenant at the time they entered the covenant (e.g., as grantor and grantee). Thus, it does NOT exist between the original parties to a real covenant absent such a relationship, nor does it exist generally between a party burdened under a real covenant and any party seeking to enforce the covenant. In contrast with horizontal privity, vertical privity refers to the relationship between an original party to a real covenant and her successor in interest. For the burden of a covenant to run, this element is satisfied if the successor holds the entire durational interest held by the covenantor at the time she made the covenant.

If L leases property to T, and L subsequently assigns L's interest to L2, whom may T hold liable when X, a paramount title holder, ejects T? response - correct A Neither L nor L2 B L2 only C L or L2 D L only

C. If L leases property to T, and L subsequently assigns L's interest to L2, T may hold L or L2 liable when X, a paramount title holder, ejects T. A landlord may assign the rents and reversion interest that he owns. The assignee is liable to the tenants for performance of all covenants made by the original landlord in the lease, provided that those covenants run with the land. The original landlord also remains liable on all of the covenants he made in the lease. X's evicting T from the entire leased premises breaches the covenant of quiet enjoyment, which runs with the land. Thus, L and L2 are personally liable to T. L only is incorrect because L2, the assignee, is liable for all lease covenants that run with the land, and the covenant of quiet enjoyment runs with the land. L2 only is incorrect because L, the original landlord, also remains liable on all covenants in the original lease after assignment. Neither L nor L2 is incorrect because the original landlord (L) remains liable on all covenants in the original lease after assignment, and the assignee (L2) is liable for all lease covenants that run with the land, including the covenant of quiet enjoyment.

If a deed never was delivered, but the listed grantee discovers the deed and records it, the deed is: A Voidable B Valid C Void

C. If a deed never was delivered, but the listed grantee discovers the deed and records it, the deed is void. A deed is not effective unless it has been delivered and accepted. While proper acknowledgment and recording raises the presumption that a deed is authentic and was validly delivered, this presumption is rebuttable in most states. Recording a deed, by itself, does not validate any aspect of an otherwise invalid conveyance (e.g., a forged or undelivered deed). Here, the grantor never delivered the deed; thus, it is not valid (i.e., void). A void deed will be set aside by a court even if title purportedly has passed to a bona fide purchaser. If a deed never was delivered, but the listed grantee discovers the deed and records it, the deed is not voidable. In contrast with void deeds, voidable deeds will be set aside only if the property has not passed to a bona fide purchaser. Deeds considered voidable include those executed by incapacitated persons (e.g., minors) or obtained through fraud in the inducement, duress, undue influence, mistake, and breach of fiduciary duty. QUESTION ID: P0111A Additional Learning

Which of the following statements is correct regarding covenants against assignment or sublease? response - incorrect A If a tenant transfers her interest in violation of a covenant against assignment or sublease, the transfer is void. B A covenant against assignment prevents the tenant from subleasing her interest. C If a landlord consents to one transfer that violates a covenant against assignment or sublease, he waives his right to avoid future transfers. D A covenant against assignment or sublease is an unreasonable restraint on alienation.

C. If a landlord consents to one transfer that violates a covenant against assignment or sublease, he waives his right to avoid future transfers. This is the Rule in Dumpor's Case. The landlord may reserve the right to avoid future transfers, but such reservation must take place at the time of granting consent. A covenant against assignment or sublease is NOT an unreasonable restraint on alienation. All jurisdictions permit and enforce such covenants. A covenant against assignment does NOT prevent the tenant from subleasing her interest. Covenants against assignment or sublease are strictly construed against the landlord. Thus, a covenant prohibiting assignment does not prohibit subleasing and vice versa.

In which of the following situations must the tenant continue to pay a portion of the rent? response - incorrect A The landlord takes possession of an unused barn on the leased premises and stores farm equipment in it. B A paramount title holder obtains a judgment in an ejectment action against the tenant. C A paramount title holder takes possession of an unused barn on the leased premises and stores farm equipment in it. D The landlord obtains a judgment in an ejectment action against the tenant.

C. If a paramount title holder takes possession of an unused barn on the leased premises and stores farm equipment in it, the tenant must continue to pay a portion of the rent. Every lease contains an implied covenant that neither the landlord nor someone with paramount title will interfere with the tenant's quiet enjoyment and possession of the premises. This covenant is breached by the tenant's total or partial actual eviction from the leased premises. Total actual eviction occurs when the landlord or a paramount title holder excludes the tenant from the entire leased premises. This terminates the tenant's obligation to pay rent. Partial actual eviction occurs when the tenant is excluded from only part of the leased premises. Partial eviction by the landlord relieves the tenant of the obligation to pay rent for the entire premises, even though the tenant continues in possession of the remainder of the premises. Partial eviction by a paramount title holder results in an apportionment of rent; i.e., the tenant is liable for the reasonable rental value of the portion that he continues to possess. A paramount title holder's taking possession of an unused barn constitutes partial actual eviction. Thus, rent will be apportioned.

Which of the following transfers creates an assignment of the lease from T to T2? response - incorrect A Four years into a six-year tenancy for years, T orally transfers his entire interest to T2 for two years. B Two years into a four-year tenancy for years, T "assigns my entire interest to T2 for one year". C One year into a five-year tenancy for years, T transfers his interest "to T2 for four years; however, if T2 breaches the original lease terms, T may reenter and retake the premises". D Six months into a tenancy at will, T transfers "my entire interest to T2".

C. If one year into a five-year tenancy for years, T transfers his interest "to T2 for four years; however, if T2 breaches the original lease terms, T may reenter and retake the premises," the effect of the transfer is an assignment of the lease from T to T2. A complete transfer of the entire remaining lease term constitutes an assignment of the lease. If the tenant retains any part of the remaining lease term, other than a right of reentry for breach of the original lease terms, the transfer is a sublease. Here, T transferred the remaining four years of the lease to T2. By the slight majority view, T's reservation of a right of reentry does not result in a sublease, but rather is still an assignment. If six months into a tenancy at will, T transfers "my entire interest to T2," the attempted assignment is void and terminates the tenancy at will by operation of law. A tenancy at will is a leasehold estate that is terminable at the will of either the landlord or the tenant. Such a tenancy terminates by operation of law if: 1. Either party dies; 2. The tenant commits waste; 3. The tenant attempts to assign his tenancy; 4. The landlord transfers her interest in the property; or 5. The landlord executes a term lease to a third person. If two years into a four-year tenancy for years, T "assigns my entire interest to T2 for one year," the effect of the transfer is to create a sublease between T and T2. The label given to the transfer by the parties does not determine whether the transfer is an assignment or a sublease. The nature of the transaction is determined by what interest, if any, the tenant retains. Here, although T "assigned" his interest to T2, he transferred only one of the remaining two years of the lease. Thus, the transfer is a sublease rather than an assignment. If four years into a six-year tenancy for years, T orally transfers his entire interest to T2 for two years, the attempted assignment is ineffective under the Statute of Frauds. Most states require that a lease creating a tenancy for more than one year, including an assignment of an interest in a lease for more than one year, be in writing to satisfy the Statute of Frauds.

In most states, the reservation of an annual rent, payable monthly, in a lease with no set termination date creates a: A Tenancy at will B Tenancy for years C Year-to-year periodic tenancy D Month-to-month periodic tenancy

C. In most states, the reservation of an annual rent, payable monthly, in a lease with no set termination date creates a year-to-year periodic tenancy . A periodic tenancy is a tenancy that continues from period to period until terminated by proper notice by either the landlord or the tenant. It may be created by implication if a lease with no set termination date provides for the payment of periodic rent. The majority view is that a lease at an annual rent, payable monthly, creates a periodic tenancy from year to year, and not a month-to-month periodic tenancy. A tenancy at will is a tenancy that continues only until the landlord or the tenant gives notice and time to quit. Although a tenancy at will can arise when a lease has no set termination date, a provision requiring annual rent payments will convert it to a periodic tenancy. A tenancy for years is a tenancy that continues for a fixed period of time and then ends automatically on its termination date. A lease with no stated duration is not a tenancy for years.

Which of the following would charge a purchaser of realty with inquiry notice? A Her grantor's use of a quitclaim deed B An unrecorded instrument that refers to a prior transaction C The fact that her grantor's deed is unrecorded

C. The fact that her grantor's deed is unrecorded would charge a purchaser of realty with inquiry notice. Inquiry notice means that a subsequent grantee is held to have knowledge of any facts that a reasonable inquiry would have revealed, even if he made no inquiry. When a grantor's deed is unrecorded, the grantee is expected—at her peril—to demand a viewing of her grantor's title documents at the time of the purchase and insist that they be recorded. Her grantor's use of a quitclaim deed would not charge a purchaser of realty with inquiry notice. A quitclaim deed releases whatever interest a grantor might have in the property and contains no covenants for title. Nonetheless, in the majority of states, grantees are not charged with inquiry notice from the mere fact that a quitclaim deed was used. An unrecorded instrument that refers to a prior transaction would not charge a purchaser of realty with inquiry notice. If a recorded instrument makes reference to an unrecorded transaction, the grantee is bound to make inquiry to discover the nature and character of the unrecorded transaction. However, this duty to inquire does not extend to situations in which an unrecorded instrument refers to the prior transaction (unless the purchaser has actual knowledge of it). Note, however, that the purchaser is charged with inquiry notice when her grantor's deed is unrecorded, as explained above. QUESTION ID: P0116A Additional Learning

A grantor executed and delivered a deed to his son conveying his land as follows: "To my son for life, but if my son dies survived by his spouse and children, then to my son's spouse for life, with the remainder in fee simple to my son's children." A year later, the son died survived by his spouse and two offspring, a girl and a boy. The boy died intestate two days after the son, leaving one child as his only heir. The common law Rule Against Perpetuities is unmodified in the jurisdiction. What are the respective interests of the spouse, the girl, and the child in the land? A The spouse has a life estate, the girl has an absolutely vested remainder, and the child has nothing. B The spouse has fee simple ownership of the land, and the girl and the child have nothing. C The spouse has a life estate, and the girl and the child have absolutely vested remainders. D The spouse has a life estate, and the girl has a vested remainder subject to open.

C. The spouse has a life estate, the girl has an absolutely vested remainder, and the child, by intestate succession, will inherit the boy's absolutely vested remainder. The remainder to the son's children was vested subject to open upon the birth of his first child. Because the son cannot have any more children after his death, all members of the class are ascertained at that time and the remainder becomes indefeasibly vested. Because the grant was to the son's "children" rather than "issue" or "descendants," there is no unborn child problem. (A) is wrong because the boy's vested remainder is inheritable by the child. (B) is wrong because the spouse has only a life estate--not a fee simple absolute--and the girl and the child have absolutely vested remainders. (D) is wrong because the class gift in the limitation "with the remainder in fee simple to my son's children" closes on the son's death; no children thereafter can be born to him, which precludes the remainder's being "subject to open."

Which of the following is not required for the burden of an equitable servitude to run to successors in interest? response - incorrect A The covenant touches and concerns the land. B The successor in interest has notice of the covenant if she has given value. C There is vertical privity between the covenantor and his successor in interest. D The covenanting parties intended that successors in interest be bound by the covenant

C. Vertical privity between the covenantor and his successor in interest is not required for the burden of an equitable servitude to run to successors in interest. An equitable servitude is a covenant (i.e., a promise to do or not to do something on the land) that, regardless of whether it runs with the land at law, can be enforced in equity against assignees of the burdened land who have notice of the covenant. The burden of an equitable servitude will run to successors in interest if: 1. The covenanting parties intended that successors in interest be bound by the covenant; 2. The successor in interest has notice of the covenant (if she has given value); and 3. The covenant touches and concerns the land (i.e., it benefits the covenantor and his successor in their use and enjoyment of the burdened land). Horizontal privity between the original covenanting parties and vertical privity between the covenantor and his successor in interest are not required.

Which of the following generally occurs when a mortgagee transfers a promissory note without a written assignment of the mortgage? A The mortgagee retains the rights to the mortgage. B The mortgage is separated from the obligation on the note. C The mortgage follows the note. D The mortgage is extinguished.

C. When a mortgagee transfers a promissory note without a written assignment of the mortgage, generally the mortgage follows the note. A mortgage is a security interest in real estate that secures an obligation, usually a promise to repay a loan, which is represented by a promissory note. The debtor (i.e., the mortgagor) gives the mortgage and the note to the lender (i.e., the mortgagee). The mortgagee who transfers her interest usually does so by indorsing the note and executing a separate assignment of the mortgage. While it is possible to transfer the note without the mortgage, the mortgage automatically will follow the properly transferred note. No separate written assignment of the mortgage is necessary. The mortgagee does NOT retain the rights to the mortgage when she transfers the note without a written assignment of the mortgage unless she expressly reserves the rights, which there would rarely be any reason for her to do. Generally, the mortgage follows the note; the mortgage is NOT separated from the obligation on the note, and the mortgage is NOT extinguished.

A court is least likely to reform a deed that __________. A Reflects the parties' mutual mistake B Reflects one party's mistake that was induced by the other party's misrepresentation C Contains a drafting error D Contains a prior agreement the parties wish to change

D. A court is least likely to reform a deed that contains a prior agreement the parties wish to change. Reformation is an equitable action in which the court rewrites a deed to make it conform to the parties' intention. Reformation will be granted if the deed does not express what the parties agreed to for any of the following reasons: 1. It reflects the parties' mutual mistake;2. It contains a drafting error; or3. It reflects one party's mistake—but only if that mistake was induced by the other party's misrepresentation or inequitable conduct. However, if property has passed to a bona fide purchaser for value who relied on the original language of the deed, the court will not reform it. QUESTION ID: P0107B Additional Learning

A grantor who conveys a fee simple determinable retains: A A right of entry B A reversion C No interest D A possibility of reverter

D. A grantor who conveys a fee simple determinable retains a possibility of reverter. A conveyance from "O to A for so long as/while/during/until [event]" creates a fee simple determinable in A and a possibility of reverter in O. A right of entry is the future interest retained by a grantor who conveys a fee simple subject to a condition subsequent. A conveyance from "O to A upon condition that/provided that/but if/if it happens that [event], then O or her heirs may enter and terminate the estate" creates a fee simple subject to a condition subsequent in A and a right of entry in O. A reversion is the future interest retained by a grantor who conveys a lesser estate than the grantor owns. Where O has a fee simple, a conveyance from "O to A for life" creates a life estate in A and a reversion in O.

A recording act that provides: "Any conveyance of an interest in land, other than a lease for less than one year, shall not be valid against any subsequent purchaser for value, without notice thereof, unless the conveyance is recorded," is a: A race statute B race-notice statute C statute of frauds D notice statute

D. A recording act that provides: "Any conveyance of an interest in land, other than a lease for less than one year, shall not be valid against any subsequent purchaser for value, without notice thereof, unless the conveyance is recorded," is a notice statute. Under a notice statute, a later purchaser of land will prevail over an earlier grantee if she takes without actual or constructive (e.g., record) notice of the earlier grant. The above language is not a race-notice statute. An example of a race-notice statute is: "Any conveyance of an interest in land, other than a lease for less than one year, shall not be valid against any subsequent purchaser for value, without notice thereof, whose conveyance is first recorded." Under a race-notice statute, a later purchaser will prevail over an earlier grantee only if she takes without actual or constructive (e.g., record) notice of the earlier grant and records before he does. The above language is not a pure race statute. An example of a pure race statute is: "Any conveyance of an interest in land, other than a lease for less than one year, shall not be valid against any subsequent purchaser whose conveyance is first recorded." Under a race statute, notice is irrelevant. The first party to record, regardless of the date of her conveyance, wins. The Statute of Frauds is not a recording act. Every conveyance of an interest in land with a duration long enough to bring into play a particular state's Statute of Frauds (typically one year) must be evidenced by a writing, signed by the party to be charged.

If a deed containing each of the following descriptions of land results in an ambiguity, to which will a court afford the most weight? A "10 acres of land known as Blackacre" B "East 45 feet to Grace Street" C "North 25 degrees to Main Street" D "From Moss Lake to the oak tree"

D. If a deed containing each of the following descriptions of land results in an ambiguity, a court will afford the most weight to the description "from Moss Lake to the oak tree." A deed must identify the land to be conveyed. When a mistake or inconsistency in the description leaves in doubt the exact location of the property, and the court otherwise lacks clear evidence of the parties' intent, the following rules of construction apply: 1. Natural monuments (e.g., "from Moss Lake to the oak tree") prevail over all other methods of description, including artificial monuments, courses and distances, and quantity measurements;2. Artificial monuments (e.g., stakes, buildings) prevail over all but natural monuments;3. Course measurements (e.g., "North 25 degrees to Main Street") prevail over distance measurements (e.g., "East 45 feet to Grace Street"); and4. All of the foregoing prevail over general descriptions such as name or quantity (e.g., "10 acres of land known as Blackacre").

If a landlord's breach of duty renders the premises unsuitable for occupancy, under the doctrine of constructive eviction, the tenant may: A Sue for breach only if the lease contained an express covenant for quiet enjoyment B Remain in possession of the premises, continue to pay rent, and sue for damages C Remain in possession of the premises and refuse to pay rent until the interference ceases D Vacate the premises, terminate the lease, and sue for damages

D. If a landlord's breach of duty renders the premises unsuitable for occupancy, the tenant may vacate the premises, terminate the lease, and sue for damages. Under the doctrine of constructive eviction, if the landlord's breach (i.e., doing an act or failing to provide some service that he has a legal duty to provide) makes the premises untenantable, the tenant may terminate the lease and also may seek damages if the following conditions are met: (i) The breach must be by the landlord or by persons acting for him. (ii) The breach must substantially and materially deprive the tenant of her use and enjoyment of the premises (e.g., flooding, absence of heat in winter). (iii) The tenant must give the landlord notice and a reasonable time to repair. (iv) The tenant must vacate the premises within a reasonable time. Because a tenant cannot claim a constructive eviction unless and until she vacates the premises, her remedies do not include remaining in possession of the premises and refusing to pay rent until the interference ceases or continuing to pay rent and suing for damages. The tenant is not limited to suing for breach only if the lease contained an express covenant for quiet enjoyment. Every lease contains an implied covenant that neither the landlord nor someone with paramount title will interfere with the tenant's quiet enjoyment and possession of the premises. If a landlord does so, the tenant has the remedies discussed above.

In general, a party who fails to tender performance on the closing date: A Is excused from performance B Has no liability for even incidental damages C Is in total breach and loses her right to enforce the contract D Has a reasonable time after the closing date to tender performance

D. In general, a party who fails to tender performance on the closing date has a reasonable time after the closing date to tender performance and avoid breach. Generally, the time of performance stated in a land sale contract is not absolutely binding. A party, even though late in tendering her own performance, can still enforce the contract if she tenders within a reasonable time after the stated date. Courts presume that time is not of the essence. However, this presumption may be overcome if: (i) The contract states that time is of the essence; (ii) The circumstances indicate that the parties intended that time is of the essence; or (iii) One party notifies the other within a reasonable time before the closing date that time is of the essence. If time is of the essence, a party who fails to tender performance on the closing date is in total breach and loses her right to enforce the contract . However, even if time is not of the essence, a party who is late in tendering performance is NOT excused from performance absent repudiation or impossibility, and will be liable for incidental damages (e.g., additional mortgage interest or taxes).

A grantor hands a deed to a grantee and says, "I want you to hold on to this deed so you can have my land when I die." Will a court likely rule that delivery occurred? A Yes, because delivery is presumed if a deed is handed to the grantee B No, because the grantor's statement is admissible to show that delivery of the deed was conditional C Yes, because the grantor's statement is not admissible to show that the deed was not delivered D No, because the grantor's statement is admissible to show that the deed was not delivered

D. No, a court will not likely rule that delivery occurred, because the grantor's statement IS admissible to show that the deed was not delivered. A deed is not effective to transfer an interest in realty unless it has been delivered. This delivery requirement is satisfied through words or conduct evidencing the grantor's intent that that title pass immediately and irrevocably, even if the right of possession is postponed until some future time. Parol evidence is admissible to prove that the grantor lacked this intent. Here, the grantor merely wanted the grantee to "hold on to this deed" but indicated that title should not pass until the grantor died. The grantor's statement is NOT admissible to show that delivery of the deed was conditional. Although parol evidence is admissible to show that no delivery was intended, if a deed is unconditional on its face and given directly to the grantee, parol evidence is inadmissible to show that the delivery was in fact subject to a condition. Although delivery is presumed if a deed is handed to the grantee, the presumption may be rebutted by, for example, evidence that the grantor did not intend the deed to have a present operative effect. QUESTION ID: P0118A Additional Learning

A landlord entered into a 10-year lease of a building with an auctioneer, who planned to use the building itself for a storage area and the covered porch at the front of the building for auctions. A term in the auctioneer's lease stated, "Lessor agrees to maintain all structures on the property in good repair." Four years into the lease, the landlord sold the property to a buyer. The buyer did not agree to perform any obligations under the lease. As instructed, the auctioneer began paying rent to the buyer. In the fifth year of the lease, the porch roof began to leak. Citing the lease terms, the auctioneer asked the buyer to repair the roof. He continually refused to do so. The auctioneer finally repaired the roof herself at a cost of $2,000. The auctioneer then brought an appropriate lawsuit to recover the money. Absent any other facts, what is the auctioneer likely to recover? response - incorrect A $2,000 from the landlord only, because the sale of the property did not sever his obligation to the auctioneer. B $2,000 from the buyer only, because a covenant to repair runs with the land. C $1,200 from the buyer and $800 from the landlord, because that represents their pro rata shares. D $2,000 from either the buyer or the landlord, because they are both in privity with the auctioneer.

D. The auctioneer may recover the cost of repair from either the landlord or the buyer. A landlord's promise in a lease to maintain the property does not terminate because the property is sold. Although no longer in privity of estate, the original landlord and tenant remain in privity of contract, and the original landlord remains liable on the covenant unless there is a novation. A novation substitutes a new party for an original party to the contract. It requires the assent of all parties and completely releases the original party. Because neither the auctioneer nor the buyer has agreed to a novation, the landlord remains liable for the covenant because he and the auctioneer remain in privity of contract even after the sale. Thus, the promise to repair can be enforced against the landlord. When leased property is sold, the purchaser may be liable for his predecessor's promises if the promise runs with the land. A covenant in a lease runs with the land if the parties to the lease so intend and the covenant touches and concerns the land. Generally, promises to do a physical act, such as maintain or repair the property, are considered to run with the land. Thus, the buyer is liable because he is in privity of estate with the auctioneer and the covenant to repair runs with the land. Consequently, both the landlord and the buyer are potentially liable to the auctioneer for the repairs. While it is true that the sale/assignment to the buyer did not sever the landlord's obligation to the auctioneer, as explained above, the landlord is not the only person who is liable to the auctioneer. Because both the landlord and the buyer are potentially liable for the repairs, (A) is incorrect. (B) is incorrect because although it is true that a covenant to repair touches and concerns the land and runs with it on assignment, the landlord as well as the buyer can be held liable. (C) is incorrect because the auctioneer may recover the full amount from either the landlord or the buyer.

A landowner died, validly devising her land "to my son for life, then to my brother and sister in fee simple." Without obtaining the brother and sister's consent, the son borrowed $20,000 from a bank, secured by a mortgage on the land, to make improvements to the land. Five years later, the son died. The brother and sister took possession of the land, but failed to make any mortgage payments. If the bank sues to recover the delinquent payments, the court should render judgment for: A The bank, because a life tenant is obligated to make repairs. B The bank, because the remaindermen are obligated to pay the principal of a debt. C The brother and sister, because the son committed ameliorative waste. D The brother and sister, because the mortgage does not encumber the fee simple.

D. The court should render judgment for the brother and sister because the mortgage does not encumber the fee simple. A life tenant is entitled to all the ordinary uses and profits of the land, which includes encumbering his own interest, but he cannot lawfully do any act that would injure the interests of the remaindermen. Permissive waste occurs when the life tenant allows the land to fall into disrepair or fails to take reasonable measures to protect the land. Although a life tenant is obligated to pay interest on any encumbrances of the fee simple estate to the extent of the income or profits from the land (or in their absence to the extent of the reasonable rental value of the land), he is liable for both principal and interest payments if the encumbrance is on the life estate alone. Here, because the son did not obtain the consent or joinder of the remainderman when he mortgaged the land, the mortgage attaches only to the life estate. Thus, the remaindermen (the brother and sister) are not liable for the mortgage payments. (A) is incorrect because the son made improvements rather than repairs to the land. Although a life tenant is obligated to preserve the land and structures in a reasonable state of repair, to the extent of the income or profits from the land (or in their absence to the extent of the reasonable rental value of the land), he is under no obligation to make permanent improvements on the land. (B) is incorrect because remaindermen are obligated to pay the principal of a debt only if the debt encumbers the entire fee simple estate. Here, only the life estate is encumbered. (C) is incorrect because ameliorative waste, which occurs when the use of the property is substantially changed but the change increases the value of the property, may have been an appropriate cause of action for the brother and sister to bring against the son for the improvements, but it does not affect the bank's rights.

A landowner owned a large tract of land, which he divided into two parcels. The northern parcel abutted a public highway. The shortest route from the southern parcel to the highway was over a private road that crossed the northern parcel. The other route was over a single-lane dirt and gravel path that wound for over four miles through the woods. The landowner sold the southern parcel to a developer, including an express easement in the private road across the northern parcel. The landowner knew of the developer's plans to open an inn on the property. The developer built the inn but never opened it to the public. Fifteen years later, the developer sold the southern parcel to an investor, who planned to open the inn to the public. The developer had never properly recorded her deed to the land, but the investor promptly recorded her deed, which made no mention of a right to cross the northern parcel via the private road. About a week after the investor took possession of the southern parcel, she learned of the provision in the developer's deed to the land. However, the landowner refuses to grant the investor permission to use the road across his property to reach the highway. Does the investor have a right to cross the northern parcel? response - incorrect A No, because the easement is not mentioned in the investor's deed, and the developer's deed containing the easement was not recorded. B No, because the investor's opening of the inn would increase the use of the easement. C Yes, but only if the developer exercised her right to use the easement when she owned the southern parcel. D Yes, even if the developer never exercised her right to use the easement when she owned the southern parcel.

D. The investor has an easement to cross the northern parcel even if the developer never exercised her right to use the easement. The original easement granted to the developer was an easement appurtenant, the benefit of which passes with a transfer of the benefited land. An easement is deemed appurtenant when the right of special use benefits the easement holder in her physical use or enjoyment of another tract of land. The land subject to the easement is the servient tenement, while the land having the benefit of the easement is the dominant tenement. The benefit of an easement appurtenant passes with transfers of the benefited land, regardless of whether the easement is mentioned in the conveyance. All who possess or subsequently succeed to title to the dominant tenement are entitled to the benefit of the easement. The easement granted to the developer was an easement appurtenant because the right to use the private road across the northern parcel (the servient tenement) benefited the developer in her use and enjoyment of the southern parcel (the dominant tenement) by providing her with the most convenient access to the public highway. Thus, when the developer sold the benefited land to the investor, the benefit of the easement also passed to the investor as an incident of possession of the southern parcel. (A) is incorrect because, as explained above, this benefit passed to the investor despite the fact that the deed to the investor made no mention of the easement. The failure to record does not affect the validity of the easement. Recordation is not essential to the validity of a deed, but only serves to protect the interests of a grantee against subsequent purchasers. Here, the dispute is between the original grantor and the successor of the original easement holder. The purpose of most recording statutes is to provide notice to a burdened party. The person who granted the easement is in no need of notice. The only relevance of recording in this situation is with respect to the servient tenement, the northern parcel. The grant of easement should be recorded on the northern parcel, or bona fide purchasers from the landowner will take free of it. However, no such purchasers are involved in this question. (B) is incorrect because the investor's use of the easement would not be a change in its use. This choice goes to the scope of the easement. The key for determining the scope is the reasonable intent of the original parties, including the reasonable present and future needs of the dominant tenement. Here, because the landowner knew of the developer's plans to open an inn, he knew that she and her guests would use the road across the northern parcel. The investor's use of the easement would be the same—her use and that of her guests. This is not a change in intended use sufficient to allow the landowner to legally prevent the investor's use of the easement. (C) is incorrect because nonuse does not extinguish an easement. Abandonment, which does terminate an easement, requires a physical act by the easement holder that manifests an intent to permanently abandon the easement (e.g., erecting a building that blocks access to an easement of way). Because there is no indication of such an act by the developer, the easement continues to benefit the southern parcel even if the developer never used it.

A landowner owned a large piece of property containing an inn and a bakery. She entered into a contract to sell the property to a purchaser for $1 million. The contract was recorded. The purchaser gave the landowner $200,000 as earnest money. The closing date was set for September 10, two months after the signing of the contract. On August 10, an arsonist set fire to the inn, which burned to the ground. On September 10, the landowner appeared at the closing and tendered the deed to the property. The buyer refused to tender the remaining $800,000 of the purchase price and demanded the return of his earnest money. The landowner sued the buyer for specific performance of the contract. The buyer countersued for the return of his earnest money. Both parties stipulate that the value of the property without the inn is $600,000, that insurance on the property had lapsed, and that the common law, unmodified by statute, applies. What is the most likely result at trial? A The landowner will not prevail on the issue of specific performance, but will be allowed to keep the earnest money. B The landowner will not prevail on the issue of specific performance and will be ordered to return the earnest money. C The landowner will prevail on the issue of specific performance, but the price will be abated to $600,000. D The landowner will prevail on the issue of specific performance for the full contract price.

D. The landowner will succeed in her suit for specific performance at the full contract price. Where property subject to a contract for sale is destroyed without the fault of either party before the date set for closing, the rule in the absence of a statute is that the risk of loss is on the buyer. Thus, the buyer must pay the contract price despite a loss due to fire, unless the contract provides otherwise. Here, the inn was destroyed by fire after the landowner and the buyer entered into their contract for the sale of the property, but before the closing date. The contract apparently was silent regarding the risk of loss and there is no applicable statute. Thus, under the common law rule, the risk of loss is on the buyer. As a result, the landowner is entitled to receive specific performance of the contract, meaning that the buyer must pay the full contract price. (A) and (B) are incorrect because they conclude that the landowner is not entitled to specific performance. As explained above, the landowner is entitled to specific performance because the risk of loss is on the buyer. (B) is also incorrect because it states that the landowner must refund the earnest money. The landowner is entitled to the full contract price; thus, there is no reason for her to return the earnest money. (C) is incorrect because it allows the buyer to tender less than the full contract price. With the buyer bearing the risk of loss, he must pay the $1 million contract price despite the decrease in the property's value due to the fire.

A rancher entered into a written contract to buy a farm from a farmer for $100,000. The contract stipulated for closing on September 30. In addition, the contract contained the following provision: "The taxes shall be prorated as agreed to by the parties at a later date." Upon the signing of the contract, the rancher gave the farmer a check for $10,000 as a down payment. On September 28, the rancher notified the farmer that he would not be able to close on the farm until October 2, because the closing on his current home, the proceeds from which were to be applied to his purchase of the farm, was unavoidably delayed due to his buyer's illness. Meanwhile, the farmer had difficulty finding a home she liked as well as the farm. She decided that she would rather not sell the farm and wished to avoid the contract with the rancher. On October 2, the rancher showed up at the closing with the $90,000 to tender to the farmer. The farmer did not show up. The rancher sues for specific performance. In whose favor will the court most likely rule? A The farmer, because the tax provision is an essential term of the contract, and it is not specific enough to satisfy the Statute of Frauds. B The farmer, because the rancher materially breached by not tendering performance on September 30. C The rancher, because of the operation of the doctrine of equitable conversion. D The rancher, because time was not of the essence.

D. The rancher will prevail because there is no evidence that time was of the essence. In general, courts presume that time is not of the essence in real estate contracts. Thus, the closing date stated in the contract is not absolutely binding in equity, and a party, even though late in tendering his own performance, can still enforce the contract if he tenders within a reasonable time. (One to two months is usually considered reasonable.) Time will be considered of the essence only if: (i) the contract so states, (ii) the circumstances indicate it was the parties' intention, or (iii) one party gives the other notice that he desires to make time of the essence. The contract in this case made no mention that time was of the essence. The facts do not indicate any circumstances, such as rapidly fluctuating prices or the need for the money to close another critical transaction, that would indicate that the rancher and the farmer intended time to be of the essence. The farmer did not give the rancher reasonable notice before September 30 that she wanted to make time of the essence. Thus, the court will not find that time is of the essence here. Because time is not of the essence, the rancher is not in material breach and is entitled to specific performance. (A) is wrong because the Statute of Frauds is not violated here. Contracts for the sale of land must be in writing to be enforceable. The essential terms for purposes of the Statute of Frauds are: the description of the property, the identification of the parties, and the price. The tax provision is not an essential term. It is an incidental matter, which need not appear in writing or even be agreed upon. (B) is wrong because, as discussed above, the rancher is not in material breach. Time was not of the essence, so the fact that the rancher did not tender his performance on September 30 did not constitute a breach of the land sale contract. (C) is wrong because the doctrine of equitable conversion will not affect the rights of the parties in this situation. The doctrine of equitable conversion holds that once an enforceable contract of sale is signed, the purchaser's interest is real property, and the seller's interest (the right to proceeds) is personal property. This is important with respect to which party bears the risk of loss if the property is damaged before the date set for closing or if one of the parties dies prior to closing. It has no effect in situations like this one where the question in issue is the enforceability of the contract itself.

On April 15, a seller entered into a valid written agreement to sell her home to a buyer for $175,000. The provisions of the agreement provided that closing would be at the buyer's attorney's office on May 15, and that the seller would deliver to the buyer marketable title, free and clear of all encumbrances. On the date of closing, the seller offered to the buyer the deed to the house, but the buyer refused to go ahead with the purchase because his attorney told him that a contractor who had done work on the house had recorded a lis pendens on May 1 against the property regarding a $10,000 contract dispute he had with the seller. The seller indicated that she was unaware of the lien, but that she was willing to go ahead with the sale and set aside funds from the purchase price to cover the contractor's claim until the dispute was resolved. The buyer still refused to proceed, stating that the seller had breached the contract. If the seller brings an action against the buyer for specific performance, what is the probable result? A The buyer prevails, because the title to the property was not marketable as of the date of closing. B The buyer prevails, because an encumbrance was on the title as of the date of closing that was subject to litigation. C The seller prevails, because under the doctrine of equitable conversion, the buyer was the owner of the property when the lis pendens was recorded, and therefore it was invalid. D The seller prevails, because an implied term of their contract was that she could use the proceeds to clear any encumbrance on the title.

D. The seller will likely prevail because she is entitled to clear the encumbrance with the proceeds of the sale. In a contract for the sale of real property, the seller of the land is entitled to use the proceeds of the sale to clear title if she can ensure that the purchaser will be protected. The seller's offer to escrow the funds in this case should act as such guarantee. Thus, (A) is incorrect. (B) is incorrect because, although there will be litigation over the contract dispute, the litigation will not affect the title to the land because the contractor is claiming only money damages and not an interest in the property. (C) is incorrect because the doctrine of equitable conversion is only applicable as against the seller and the buyer, and does not affect the right of some third party with regard to attaching property held in the name of a debtor.

All of the deeds for the lots on a city block contained a restrictive covenant requiring that all houses built on the lots be set back a minimum of 50 feet from the sidewalk. Local zoning regulations required that all homes on the block be set back a minimum of 35 feet from the sidewalk. A man purchased a lot on the block on which it would be possible to build a home with a 50-foot setback. However, the man applied to the city zoning commission for a variance reducing the setback to 30 feet from the sidewalk. In his petition, the man cited the unusual shape of the lot and asserted that it would cause hardship for him to build in compliance with the 35-foot setback required by the zoning regulations. The zoning commission granted the man the variance. A woman whose home was located on the block noticed surveyors putting up ropes 30 feet from the sidewalk on the man's lot, and she discovered that the man planned to build a home with only a 30-foot setback. The woman brings suit to enjoin the man from building a residence with a setback of less than 50 feet. Who will prevail? A The man, because zoning regulations take precedence over restrictive covenants as a matter of public policy. B The man, because equity will not impose a hardship. C The woman, because the man will be unjustly enriched if he is permitted to build a 30-foot setback. D The woman, because a zoning variance does not affect the enforcement of a restrictive covenant.

D. The woman will be able to enforce the restrictive covenant as an equitable servitude. Zoning regulations and restrictive covenants in private deeds are completely separate concepts. Both must be complied with, and neither provides any excuse for violating the other. Thus, a variance from the government regulation does not prevent enforcement of the private covenant. The court will enforce the covenant as an equitable servitude because the woman is seeking an injunction. An equitable servitude is a covenant that, regardless of whether it runs with the land at law, equity will enforce against the assignees of the burdened land who have notice of the covenant. Here, all of the deeds contained the restrictive covenant. There is no indication that the man did not have notice of the restriction, and it is both possible and reasonable for him to comply with the restriction at this stage. Privity of estate is not required because the majority of courts enforce the servitude as an equitable interest in the land itself. Hence, the woman will obtain the injunction. (A) is incorrect because, as noted above, zoning regulations and covenants in deeds are completely separate; thus, the zoning regulation would not take precedence over the covenant. The only time a zoning regulation might prevent enforcement of such a covenant is where enforcement would result in a zoning violation (e.g., covenant calls for single family residential housing only, while the land is zoned strictly commercial). (B) is an incorrect statement of law. Equity will impose a hardship, although it will try to balance the hardships between the parties. Here, the hardship on the man is not unreasonable because it is possible for him to build a house that complies with the setback restriction and he has not yet dug the foundation for the house he was planning. (C) is incorrect because nothing in the facts indicates that the man will be unjustly enriched by building his home in violation of the covenant; e.g., there is no indication that his home or property will be worth more simply because it has a 30-foot setback rather than a 50-foot setback.

Which of the following is true when a seller of land dies before the contract closes? A The contract is voidable by the seller's estate. B The successors to the seller's personal property must give up equitable title at closing. C The contract is voided by the seller's death. D The successors to the seller's real property must give up legal title at closing.

D. When a seller of land dies before the contract closes, the successors to the seller's real property must give up legal title at closing. Under the doctrine of equitable conversion, the buyer of land is considered to own (i.e., hold equitable title to) the real property once the contract is signed. The seller is entitled to the proceeds of sale. Equity regards the seller as holding bare legal title in trust for the buyer as security for the debt owed. When a party to the contract dies before closing, her interest passes accordingly. The deceased seller's personal property takers are thus entitled to the sale proceeds on closing but must surrender legal title at that time. The successors to the seller's personal property do NOT give up equitable title at closing when a seller of land dies before the contract closes. As stated above, under the doctrine of equitable conversion, the buyer obtains equitable title to the land upon the signing of the contract. If the seller dies before closing, the bare legal title she held passes to the takers of her real property, who must surrender it to the buyer at closing (when both legal and equitable titles merge in the buyer). The takers of the seller's personal property succeed only to the proceeds of the sale, not the title to the real property. When a seller of land dies before the contract closes, the contract is NOT voidable by the seller's estate. It can be enforced against the takers of her real property when closing occurs. Furthermore, the contract is NOT voided by the seller's death. As is explained above, the doctrine of equitable conversion affects the passage of title when a party to a land sale contract dies before the closing. On closing, the seller's estate must surrender legal title to the buyer, and the estate is entitled to the proceeds of the sale.

A man had rented a woman's home from her for seven years. When the time came to sign a new lease, the woman decided that because the man had always been a quiet tenant, she would continue to charge him only $350 per month rent instead of the $500 to $550 she could probably get otherwise. The new lease was for a period of five years, and by its terms, the man was specifically prohibited from assigning the lease without the woman's specific written consent. About a year later, the man got married and moved into his new wife's home. Instead of giving up his lease, the man sublet the property to a friend for $500 a month. The man did not get the woman's permission to sublease the property. If the woman brings an action to either eject the friend from the premises or to recover damages from the man for subletting the premises without her consent, what is the most likely result? response - incorrect A The woman will be able to recover damages and to eject the new tenant. B The woman will be able to eject the new tenant only, because she has suffered no money damages. C The woman will not be able to eject the new tenant because, although the man did not have the right to sublet, he had the power, but she will be entitled to recover the full rent paid by the new tenant because it would be unfair to let the man profit from his wrongful act. D The woman will have no cause of action for either ejectment or damages.

D. The woman will most likely have no cause of action for either ejectment or damages. There are two ways for a tenant to transfer the right to possession under a lease: assignment (transferring the entire period of time remaining under the lease) and sublease (transferring only a portion of the time remaining under the lease). Restraints on alienation are traditionally strictly construed. Thus, a covenant prohibiting assignment does not prohibit subleasing and vice versa. Hence, this prohibition against assignment would not be read to include a prohibition against subleasing. Therefore, the woman would have no cause of action against the man, and (A) and (B) are incorrect. (C) is incorrect. If a tenant transfers (assigns or sublets) in violation of a prohibition in the lease against transfers, the transfer is not void, but the landlord usually may terminate the lease under either the lease terms or a statute. Here, however, there is no cause of action because subleasing was not prohibited.

To acquire a prescriptive easement on property, the claimant's use must be: A Under color of title B In good faith and with the owner's permission C Exclusive, open and notorious, and adverse for the statutory period D Open and notorious, adverse, and continuous for the statutory period

D. To acquire a prescriptive easement on property, the claimant's use must be open and notorious, adverse, and continuous for the statutory period. Acquiring an easement by prescription is analogous to acquiring title to property by adverse possession, except that the use need NOT be exclusive (i.e., the user may share the use with the owner or other easement claimants). There is no requirement that the use be in good faith, and adverse use means the user does NOT have the owner's permission. Moreover, the claimant's use need not be under color of title. Color of title is a document that purports to give title, but for reasons not apparent from its face, does not. Under certain circumstances, color of title may be needed to establish adverse possession, but it is not required to acquire a prescriptive easement.

Under which theory can the mortgagee take possession of the mortgaged property upon the mortgagor's default? A The title theory only B Either the lien theory or the intermediate theory C The lien theory only D Either the title theory or the intermediate theory

D. Under either the title theory or the intermediate theory, the mortgagee may take possession of the mortgaged property upon the mortgagor's default. Under the title theory, followed in a minority of states, legal title is in the mortgagee until the mortgage has been satisfied or foreclosed. Thus, the mortgagee is entitled to possession upon demand at any time, which means the mortgagee can take possession as soon as the mortgagor defaults. The same is true in the few states that follow the intermediate theory, under which legal title transfers from the mortgagor to the mortgagee on default. Under the lien theory, followed in a majority of the states, the mortgagee is deemed to hold a security interest in the land and the mortgagor is considered the owner until foreclosure. Thus, the mortgagee may not take possession of the land before foreclosure.

Which of the following is NOT required for the burden of an equitable servitude to run to successors in interest? A The covenant touches and concerns the land. B The successor in interest has notice of the covenant if she has given value. C The covenanting parties intended that successors in interest be bound by the covenant. D There is vertical privity between the covenantor and his successor in interest.

D. Vertical privity between the covenantor and his successor in interest is not required for the burden of an equitable servitude to run to successors in interest. An equitable servitude is a covenant (i.e., a promise to do or not to do something on the land) that, regardless of whether it runs with the land at law, can be enforced in equity against assignees of the burdened land who have notice of the covenant. The burden of an equitable servitude will run to successors in interest if: 1. The covenanting parties intended that successors in interest be bound by the covenant; 2. The successor in interest has notice of the covenant (if she has given value); and 3. The covenant touches and concerns the land (i.e., it benefits the covenantor and his successor in their use and enjoyment of the burdened land). Horizontal privity between the original covenanting parties and vertical privity between the covenantor and his successor in interest are not required.

Which of the following best states the result when O does not acquire title to property until after he purported to convey the property to A? A O will retain title because he lacked capacity to convey the land. B A will acquire title regardless of the terms of her deed. C A can acquire title only if she was a bona fide purchaser. D A can accept title to the land or possibly sue O for breach of covenants for title.

D. When O does not acquire title to property until after he purported to convey the property to A, A can accept title to the land or possibly sue O for breach of covenants for title. Under the doctrine of estoppel by deed, a grantor impliedly covenants that he will convey title upon its acquisition and is estopped to deny that he acquired title on the grantee's behalf. Alternatively, in many jurisdictions the grantee may opt to sue O for damages for breach of covenants for title, if any (i.e., if a general warranty deed was used), because O lacked title at the time of the conveyance. When O does not acquire title to property until after he purported to convey the property to A, O will NOT retain title because he lacked capacity to convey the land. Instead, the doctrine of estoppel by deed applies. If a grantor purports to convey an estate that he does not then own, his subsequent acquisition of title to the property automatically inures to the benefit of the grantee. When O does not acquire title to property until after he purported to convey the property to A, A will NOT acquire title regardless of the terms of her deed. Estoppel by deed applies to grantees who take by warranty deed (i.e., deeds that contain covenants for title). Additionally, many courts apply the doctrine any time the deed expressly purports to convey a particular estate, regardless of covenants for title. In most states, however, the doctrine will not be applied when the conveyance is by a quitclaim deed—i.e., a deed releasing whatever interest, if any, the grantor has in the property. When O does not acquire title to property until after he purported to convey the property to A, A will NOT acquire title only if she was a bona fide purchaser ("BFP"). A BFP is a purchaser who gives valuable consideration and lacks notice of a prior conveyance. The first grantee's BFP status is irrelevant to the application of the doctrine of estoppel by deed. QUESTION ID: P0117 Additional Learning


Ensembles d'études connexes

cse374 final exam quiz questions

View Set

৩.১ বাংলাদেশের ভূ-প্রকৃতি, অবস্থান, জলবায়ু

View Set

Lesson 25 Quiz: Advertising Rules and Regulations

View Set

UIUC Psych 238 Quiz 1 (Chs 1,2,3,4)

View Set